You are on page 1of 102

1

1. The fact that the- of confrontation is no longer as popular as it once was - progress in race relations.
A. insidiousness - reiterates
B. practice - inculcates
C. glimmer - foreshadows
D. technique - presages
E. reticence - indicates

Ans :D

2. A child should not be - as being either very shy or over - aggressive.


A. categorized
B. instructed
C. intoned
D. distracted
E. refrained

Ans :A

3. President Anwar el - Sadat of Egypt, disregarding - criticism in the Alab world and in his own
Government, - accepted prime minister Menahem Begin's invitation to visit Israel in order to address
the Israeli parliament.
A. acrimonious - formally
B. blemished - stiffly
C. categorical - previously
D. malignant - plaintively
E. charismatic - meticulously

Ans :A

4. In his usual - manner, he had insured himself against this type of loss.
A. pensive
B. providential
C. indifferent
D. circumspect
E. caustic

Ans :D

5. We never believed that he would resort to - in order to achieve his goal; we always regarded him as a
- man.
A. charm - insincere
B. necromancy - pietistic
C. logic - honorable
D. prestidigitation - articulate
E. subterfuge - honest

Ans :E

6. The Sociologist responded to the charge that her new theory was - by pointing out that it did not in
fact contradict accepted sociological principles.
A. unproven
B. banal
C. superficial
D. complex
E. heretical

Ans :E

7. Despite assorted effusion to the contrary, there is no necessary link between scientific skill and
humanism, and quite possibly, there may be something of a - between them.
2
A. dichotomy
B. congruity
C. reciprocity
D. fusion
E. generosity

Ans :E

8. The most technologically advanced societies have been responsible for the greatest - indeed
savagery seems to be indirect proposition to -
A. inventions - know-how
B. wars - viciousness
C. triumphs - civilizations
D. atrocities - development
E. catastrophes - ill-will

Ans :D

9. Ironically, the party leaders encountered no greater - their efforts to build as Progressive Party than
the - of the progressive already elected to the legislature.
A. obstacle to - resistance
B. support for - advocacy
C. praise for - reputation
D. threat to - promise
E. benefit - success

Ans :A

10. The simplicity of the theory - its main attraction - is also its - for only by - the assumptions of the
theory is it possible to explain the most recent observations made by researchers.
A. glory - rejecting
B. liability - accepting
C. undoing - supplementing
D. downfall - considering
E. virtue - qualifying

Ans : C

11. That the Third Battalion's fifty percent casually rate transformed its assault on Hill 306 from a
brilliant stratagem into a debacle does not - eyewitness reports of its commander's extra-ordinary -
in deploying his forces.
A. invalidate - brutality
B. gainsay - cleverness
C. underscore - ineptitude
D. justify - rapidity
E. corroborate -determination

Ans : B

12. No longer - by the belief that the world around us was expressly designed for humanity, many
people try to find intellectual - for that lost certainty in astrology and in mysticism.
A. satisfied - reasons
B. reassured - justifications
C. restricted - parallels
D. sustained - substitutes
E. hampered - equivalents

Ans : D
3
13. In eighth-century Japan, people who - wasteland were rewarded with official ranks as part of an
effort to overcome the shortage of - fields.
A. cultivated - domestic
B. located - desirable
C. conserved - forested
D. reclaimed - arable
E. irrigated - accessible.

Ans :D

14. Clearly refuting sceptics, researchers have - not only that gravitational radiation exists but that it
also does exactly what the theory- it should do.
A. assumed - deducted
B. estimated - accepted
C. supposed - asserted
D. doubted - warranted
E. demonstrated - predicted.

Ans :E

15. Melodramas, which presented stark oppositions between innocence and criminality, virtue and
corruption, good and evil, were popular precisely because they offered the audience a world - of -
A. deprived - polarity
B. full - circumstantiality
C. bereft - theatricality
D. devoid - neutrality
E. composed - adversity.

Ans :D

16. Sponsors of the bill were-because there was no opposition to it within the legislative, until after the
measure had been signed into law.
A. well-intentioned
B. persistent
C. detained
D. unreliable
E. relieved.

Ans :B

17. Ecology, like economics, concerns itself with the movement of valuable - through a complex
network of producers and consumers.
A. nutrients
B. dividends
C. communications
D. artifacts
E. commodities.

Ans :C

18. Having fully embraced the belief that government by persuasion is preferable to government by - the
leaders of the movement have recently - most of their previous statements supporting
totalitarianism.
A. proclamation - codified
B. coercion - repudiated
C. participation - moderated
D. intimidation - issued
E. demonstration - deliberated.

Ans :B
4
19. It would be difficult for one so - to be led to believe that all men are equal and that we must disregard
race, color and creed.
A. tolerant
B. democratic
C. broadminded
D. emotional
E. intolerant.

Ans :E

20. Many philosophers agree that the verbal aggression of profanity in certain redical newspapers is not
- or childish, but an assault on - essential to the revolutionary's purpose.
A. insolent - sociability
B. trivial - decorum
C. belligerent - fallibility
D. serious - propriety
E. deliberate - affectation.

Ans :B

21. The - tones of the flute succeeded in - his tense nerves.


A. rhapsodic - minimising
B. blatant - enhancing
C. hovendous - calming
D. vibrant - portraying
E. mellifluous - soothing.

Ans :E

22. Without the psychiatrist's promise of confidentiality, trust is - and the patient's communication
limited; even though confidentiality can thus be seen to be precious in thercopy, moral
responsibility sometimes requires a willingness to - it.
A. lost - forget
B. implicit - extend
C. impaired - sacrifise
D. ambiguous - apply
E. assumed - examine.

Ans :C

23. Parts of seventeenth-century Chinese pleasure gardens were not necessarily intended to look -they
were designed expressly to evoke the agreeable melancholy resulting from a sense of the - of
natural beauty and human glory.
A. great - immutability
B. joyful - mortality
C. conventional - wildness
D. cheerful - transitoriness
E. colorful - abstractness.

Ans :D

24. Despite the - of many of their colleagues, some scholars have begun to emphasize ''pop culture'' as
a key for - the myths, hopes, and fears of contemporary society.
A. pedantry - reinstating
B. enthusiasm - symbolizing
C. skepticism - deciphering
D. antipathy - involving
E. discernment - evaluating.

Ans :C
5
25. If duty is the natural - of one's the course of future events, then people who are powerful have duty
placed on them whether they like it or not.
A. outgrowth - control over
B. arbiter - responsibility for
C. correlate - understanding of
D. determinant - involvement in
E. mitigant - preoccupation with .

Ans :A

26. Clearly refuting sceptics, researches have - not only that gravitational radiation exists but that it also
does exactly what the theory - it should do.
A. supposed - asserted
B. voubted -warranted
C. assumed - deduced
D. demonstrated - predicted
E. estimated - accepted

Ans :D

27. The Neolatonists' conception of a deity, in which perfection was measured by abundant fecundity,
was contradicted by that of the Aristotelians, in which perfection was displayed in the - of creation.
A. variety
B. economy
C. profusion
D. clarity
E. precision.

Ans :B

28. It is a great - to be able to transfer useful genes with as little extra gene material as possible,
because the donor's genome may contain, in addition to desirable genes, many genes with - effects.
A. Disappointment - superfluous
B. Convenience - exquisite
C. Advantage - deleterious
D. Accomplishment - profound
E. Misfortune - unpredictable.

Ans :C

29. While admitting that the risks incurred by use of the insecticide were not - the manufacturer's
spokesperson argued that effective - were simply not available.
A. indeterminable - safeguards
B. unusual - alternatives
C. inconsequential - substitutes
D. proven - antidotes
E. increasing - procedures.

Ans :C

30. Human reaction to the realm of though is often as strong as that to sensible presences; our higher
moral life is based on the fact that - sensations actually present may have a weaker influence on our
action than do ideas of - facts.
A. emotional - impersonal
B. familiar : symbolic
C. disturbing - ordinary
D. material - remote
E. defenitive - controvoisial.

Ans :D
6
31. Some scientists argue that carbon compounds play such a central role in life on earth because of
the possibility of - resulting from the carbon atom's ability to form an unending series of different
molecules.
A. variety
B. stability
C. deviations
D. invigorations
E. reproduction.

Ans :A

32. It would be difficult for one so - to be led to believe that all men are equal and that we must disregard
race, color and creed.
A. intolerant
B. democratic
C. emotional
D. patient
E. broadminded.

Ans :A

33. An occasional - remark spoiled the - that made the paper memorable.
A. colloquial
B. trite - cliches
C. urbane - sophisticated
D. hackneyed - originality
E. jovial - fun.

Ans :D

34. Broadway audiences have become inured to - and so - to be pleased as to make their ready ovations
meaningless as an indicator of the quality of the production before them.
A. cleverness : eager
B. condescension : disinclined
C. sentimentality : reluctant
D. mediocrity : desperate
E. histrionics : unlikely

Ans :D

35. Nineteenth - century scholars, by examining earlier geometric Greek art, found that classical Greek
art was not a magical - or a brilliant - blending Egyptian and Assyruin art, but was independently
evolved by Greeks in Greece.
A. conversion - annexation
B. apparition - amalgam
C. stratagem - appropriation
D. paradigm - construct
E. example - synthesis

Ans :B

36. The struggle of the generations is one of the obvious constants of human affairs; therefore, it may
be presumptuous to suggest that the rivalry between young and old in western society during the
current decade is - critical.
A. archetypally
B. perennially
C. disturbingly
D. uniquely
E. cautiously

Ans :D
7
37. Even though in today's Soviet union the - Muslim clergy have been accorded power and privileges,
the Muslim laity and the rank - and - file clergy still. Have little - to practice their religion.
A. adversaries of - inclination
B. traditionalists among - incentive
C. practitioners among - opportunity
D. leaders of - latitude
E. dissidents within -obligation

Ans :D

38. Unlike the Shakespearean plays, The ''closet dramas'' of the nineteenth century were meant to be -
rather than -
A. seen - acted
B. read - acted
C. produced - acted
D. quiet - loud
E. sophisticated - urbane

Ans :B

39. The little - known but rapidly expanding use of computers in mapmaking is technologically similar to
the more - uses in designing everything from bolts to satellites.
A. ingenuous
B. recent
C. secure
D. publicized
E. successful

Ans :D

40. Although his out numbered troops fought bravely, the general felt he had no choice but to - defeat
and - a retreat.
A. oversee - reject
B. acknowledge - order
C. hasten - suggest
D. seek - try
E. overcome - request

Ans :B

41. No hero of ancient or modern times can surpass the Indian with his lofty contempt of death and the -
with which he sustained the cruelest coffliction.
A. guide
B. assent
C. reverence
D. fortitude
E. concern

Ans :D

42. The hostess attempted to - a romantic atmosphere that would bring the two young people together
in -
A. expand - fealty
B. present - collusion
C. simulate - conflict
D. introduce - cacophony
E. contrive - matrimony

Ans :E
8
43. Employers who retire people who are willing and able to continue working should realize that - age
is not an effective - in determining whether an individual is capable of working.
A. intellectual - criterion
B. Chronological - criterion
C. Physical - barrier
D. deteriorating - value
E. chronological - factor

Ans :B

44. As the sun rose, the morning mists were borne away on the - like strands of -
A. whirlwind - flotsam
B. wind - cactus
C. morass - tundra
D. zephyr - gossamer
E. holocaust - taffeta

Ans :D

45. The playwright was known not for his original ideas that had been propounded by others.
A. rejection
B. consideration
C. invention
D. reiteration
E. plagiarism

Ans :E

46. The gypsy girl, decked out in - finery, and with her disheveled hair streaming over shoulders, was
indeed a - sight.
A. verdant - wistful
B. sartorial - flagrant
C. specious - poignant
D. tawdry - bizarre
E. opulent - debonair

Ans :D

47. Yellow fever, the disease that killed 4,000 Philadelphians in 1793, and so - Memphis, Tennessee, that
the city lost its charter, has reappeared after nearly two decades in - in the western hemisphere.
A. disabled - quarantine
B. decimated - abeyance
C. terrorized - contention
D. ravaged - secret
E. coupled - quiescence

Ans :B

48. The painting was larger than it appeared to be, for hanging in a darkened recess of the chapel, it was
- by the perspective.
A. embellished
B. improved
C. jeopardised
D. aggrandized
E. diminished

Ans :E

49. We have in America - speech that is neither American, Oxford English, nor English but a - of all
three.
9
A. motley - miracle
B. nasal - blend
C. feigned - patchwork
D. mangled - medley
E. hybrid - combination

Ans :E

50. Old beliefs die hard, even when jobs become - the long - standing fear that unemployment could
return at a moments notice -
A. protected - subsided
B. vacant - perished
C. available - receded
D. plentiful - persisted
E. easier - charged

Ans :D

51. Not only the - are fooled by pcopagandas we can all be misled if we are not -
A. people - mature
B. ignorant - cynical
C. masses - cautious
D. uncultured - concerned
E. gullible - wary

Ans :E

52. -- merciful by nature, he was - toward the murderer.


A. although - unmoving
B. while - unjust
C. truly - indicative
D. though - kind
E. albeit - implacable

Ans :E

53. When the news of his - with the enemy become known, he was hanged in -
A. collusion - effigy
B. conversation's - earnest
C. involvement - martyrdom
D. complacency - retaliation
E. bickering - response

Ans :D

54. He was so - by the interplay of the colors that varied in brilliance and pattern as the music rose and
fell, that he asked the price of the device.
A. overwrought
B. penalized
C. repelled
D. inteugued
E. penalized

Ans :D

55. The absence of a sense of outrage and grief at national tragedy is an - of moral responsibility.
A. intervention
B. energising
C. abdication
D. administration
E. actuation.
10
Ans :C

56. In an effort to - its operations, the corporation announced it was acquiring a - company in a different
type of manufacturing.
A. diversify - subsidiary
B. adumberate - solvent
C. multiply - protracted
D. intensify - fluctuating
E. establish - sequential.

Ans :A

57. Samuel Clemens chose the - Mark Twain as a result of his knowledge of river boat piloting.
A. mountebank
B. protagonist
C. misanthrope
D. hallucination
E. pseudonym.

Ans :E

58. To meet all - a source of - electrical power was added to the train's engine.
A. Integuments - parallel
B. possibilities - incidental
C. amenities - diverse
D. contingencies - auxiliary
E. conveniences - automatic.

Ans :D

59. Since the escaping vapors proved to be highly -, measures were at once taken for the - of the
experiments.
A. Volatile - ratification
B. Observable - insulation
C. Gaseous - reduction
D. Noxious - cessation
E. Incriminating - destruction.

Ans :D

60. Eric Fromm does not agree that man is - in Freudian sexual dilemmas for if the - that man creates
can be changed for the better, there is hope that the state of man can be changed as well.
A. Tortured - goals
B. Trapped - institutions
C. Caught - symbols
D. Engulfed - life
E. Confused - meanings.
61. As man reached the stars, a booming population threatened to destroy the - of life on his home
planet and even its chances for -
A. Quality - survival
B. Basis - growth
C. Existence - upliftment
D. chances - improvement
E. meaning - understanding.

Ans :A

62. Until the current warming trend exceeds the range of normal climatic fluctuations, there will be,
among scientists, considerable - the possibility that increasing levels of atmosphere Co2 can cause
long term warming effects
A. interest in
11
B. uncertainty about
C. experimentation on
D. enthusiasm for
E. worry about

Ans :B

63. Having no sense of moral obligation, shipler was as little subject to the - of conscience after he
acted as he was motivated by its - before he acted.
A. balm - eloquence
B. qualms - atonement
C. reproaches - prompting's
D. rewards - chastisement
E. ridicule - allure

Ans :C

64. Slang is a language that rolls up its sleeves, spits on its hands and -
A. goes to work
B. stays cool
C. embarrasses its user
D. communicates
E. puts its foot down

Ans :B

65. Famous among job seekers for its - , the company, quite apart from generous salaries, bestowed on
its executives annual bonuses and such - as low - interest home mortgages and company cars.
A. largesse - perquisites
B. altruism - credits
C. magnanimity - reparations
D. discernment - prerogatives
E. inventiveness - benefits

Ans :A

66. Moving and parked, the automobile devours urban land, leaving the buildings as mere - of habitable
space in a sea of dangerous and ugly traffic
A. asylums
B. remnants
C. blocks
D. mountains
E. islands

Ans :E

67. In the current research program, new varieties of apple trees are evaluated under different
agricultural - for tree size, bloom density, fruit size,- to various soils, and resistance to pests and
disease.
A. conditions - adaptability
B. configurations - propensity
C. circumstances - proximity
D. auspices - susceptibility
E. regulations - conformity

Ans :A

68. For many young people during the roaring twenties, a disgust with the excesses of American culture
- a wanderlust to provoke an exodus abroad.
A. reflected
B. stymied
C. conflicted with
12
D. overwhelmed
E. combined with

Ans :E

Verbal Section: Analogy

Directions:
Each of the GRE sample analogy questions below consists of two words that have a certain relationship to each
other, followed by five lettered pairs of related words. Select the lettered pair of words.

Following are some GRE sample analogy questions.

1. ANGLE : DEGREE
A. area : square inch
B. milk : quart
C. society : classes
D. letter : alphabet
E. time : minutes

Ans : A

2. CONFIRMED : INVETERATE
A. knowledge : supposed
B. financial : bankrupt
C. immature : callow
D. credible : incredible
E. careful: punishing

Ans :B

3. LULLABY : BARCAROLE
A. birth : marriage
B. night : morning
C. cradle : gondola
D. song : poem
E. carol : sonneteer

Ans :C

4. ZOOLOGY : ANIMALS
A. ecology : pollution
B. botany : plants
C. chemistry : atoms
D. history : people
E. mathematics : geometry

Ans :A

5. DORY : VAN
A. dairy : cow
B. fish : vehicle
C. freighter : caisson
D. runners : wheels
E. danish : Dutch

Ans : C
13
6. PARQUET : WOOD
A. color : painting
B. mosaic : glass
C. potpourri : medley
D. collage : tapestry
E. linoleum : marble

Ans : B

7. SAW : CARPENTER
A. Scissors : tailor
B. Wagon : farmer
C. Brush : painter
D. Typewriter : author
E. Trowel : bricklayer

Ans : A

8. LURK : WAIT
A. boost : elevate
B. deplete : drain
C. abscond : depart
D. bilk : cheat
E. topple : stabilize

Ans : C

9. ALCHEMY : SCIENCE
A. nostrum : remedy
B. sideshow : carnival
C. ploy : tactic
D. forgery : imitation
E. burlesque : comedy

Ans : A

10. NEEDLE : KNIT


A. bait : fish
B. match : fire
C. loom : weave
D. soap : wash
E. bed : sleep

Ans : C

11. PARENTHESIS : EXPLANATION


A. ellipsis : omission
B. asterisk : exaggeration
C. synopsis : affectation
D. apostrophe : annotation
E. synthesis : interpolation

Ans : A

12. CENSUS : POPULATION


A. manifest : debts
B. roster : audience
C. itinerary : journeys
D. inventory : merchandise
E. state : incumbents
14
Ans : D

13. STANZA : POEM


A. mimicry : pantomime
B. duet : chorus
C. act : opera
D. rhyme : verse
E. pirouette : ballet

Ans : C

14. EXHORT : SUGGEST


A. conspire : plan
B. tamper : adjust
C. crave : accept
D. goad : direct
E. instruct : teach

Ans : D

15. SAND PAPER : ABRASIVE


A. gasoline : refined
B. grativity : irritant
C. polish : floors
D. acrylic : emulsion
E. oil : lubricant.

Ans :E

16. DIAPHANOUS : CACOPHONOUS


A. translucent : transparent
B. transparent : noisy
C. sheer : opaque
D. harmonious : discordant
E. twofold : multiple.

Ans :B

17. INFANCY : SENILITY


A. january : October
B. incipient : critical
C. day : night
D. conclusion : climax
E. dawn : dusk.

Ans :E

18. RIG : CONTEST


A. solve : conundrum
B. predict : race
C. repudiate : thesis
D. gerrymander : district
E. incriminate : evidence

Ans :D

19. ARBORETUM : TREES


A. aviary : birds
B. greenhouse : garden
15
C. museum : painters
D. grove : forest
E. zoo : range

Ans :D

20. MENDICANT : IMPECUNIOUS


A. hat : askew
B. liar : poor
C. complainer : petulant
D. critic : quizzical
E. philanthrophist : prodigal.

Ans :C

21. RELAPSE : CONVALESCENCE


A. dissonance : harmony
B. feudalism : industrialization
C. repetition : monotony
D. impasse : debate
E. recidivism : rehavbilitation.

Ans :E

22. BOUQUET : FLOWERS


A. corn : husk
B. woodpile : logs
C. forest : thicket
D. mist : fog
E. drift : snow.

Ans :B

23. TRIANGLE : QUADRILATERAL


A. rectangle : octagon
B. cone : cube
C. pentagon : hexagon
D. plane : solid
E. regular : symmetrical.

Ans :C

24. SARTORIAL : TAILOR


A. thespian : designer
B. rhetorical : questioner
C. pictorial : musician
D. histrionic : singer
E. terpsichorear : dancer.

Ans :E

25. NECROMANCY : GHOSTS


A. magic : legerdemain
B. alchemy : gold
C. sorcery : spirits
D. fortune_telling : gypsies
E. romance : stories.

Ans :C
16
26. DRUM : TYMPANI
A. piano : orchestra
B. cornet : percussion
C. stick : baton
D. violin : viola
E. oboe : woodwind.

Ans :E

27. EXTROVERT : RETICENT


A. reprobate : humility
B. strategist : decisiveness
C. zealot : loyalty
D. maverick : conformity
E. renegade : ambition.

Ans :D

28. HYGROMETER : BAROMETER


A. snow : rain
B. humidity : pressure
C. water : mercury
D. temperature : weather
E. forecast : rain.

Ans :B

29. EXEMPTION : EXCLUSIONS


A. discharge : elimination
B. debarment : prevention
C. immunity : isolation
D. forgive : condone
E. enclosure : open.

Ans :C

30. FEBRILE : ILLNESS


A. classic : cultivation
B. delusional : insanity
C. eccentric : discrimination
D. tenacious : astonishment
E. juvenile : maturity.

Ans :B

31. DISAPPROBATION : CONDEMN


A. calumny : eulogise
B. enigma : enlighter
C. fallacy : diseminate
D. exhortation : urge
E. solvency : deploy.

Ans :D

32. GEM : TURQUOISE


A. lettuce : green
B. pear : orange
C. stone : magnetta
D. vine : cherry
17
E. flower : violet.

Ans :E

33. WINE : GRAPES


A. liquor : intoxicating
B. whiskey : hops
C. champagne : raisins
D. vodka : potatoes
E. vineyard : winery.

Ans :D

34. DEBATE : FORENSIC


A. concerto : harmonizing
B. drama : histrionic
C. opera : spoken
D. argument : domestic
E. novel : original.

Ans :B

35. NOISOME : GARBAGE


A. heavy : metal
B. warmth : snow
C. fragrant : incense
D. liquid : perfume
E. loud : music.

Ans :C

36. CONDUIT : WATER


A. behaviour : liquid
B. electricity : television
C. artery : blood
D. wire : sound
E. pump : oil.

Ans :C

37. BIZARRE : EXOTIC


A. wild : tame
B. lively : livid
C. stage : dancer
D. commonplace : routine
E. ordinary : exceptional.

Ans :D

38. ENTREPRENEUR : LABORER


A. mediator : conflict
B. capitalism : communism
C. profits : wages
D. arbitrator : capitalist
E. moonlighting : worker.

Ans :C

39. ANTIMACASSAR : SOFA


18
A. picture : frame
B. rug : floor
C. pillow : bed
D. door : window
E. table : chair.

Ans :B

40. NOTABLE : NOTORIOUS


A. heinous : atrocious
B. philandering : pleasant
C. philanthropic : miserly
D. nefarious : secret
E. philanthropic : benevolent.

Ans :E

41. BABBLE : TALK


A. though : blank
B. look : espy
C. wink : eye
D. leer : ogle
E. simper : smile.

Ans :E

42. ALCOVE : RECESS


A. column : entrance
B. foundation : building
C. dome : roof
D. turret : chimney
E. foyer : ballroom

Ans :C

43. FIRM : INTRANSIGHT


A. faithful : resolute
B. improvident : industrious
C. vague : inattentive
D. concerned : obsessed
E. malleable : tractable

Ans :D

44. EPAULET : SHOULDER


A. medal : chest
B. decoration : uniform
C. knapsack : back
D. sword : scabbard
E. sash : window

Ans :A

45. ANACHRONISM : CHRONOLOGY


A. tradition : custom
B. variations : incongruity
C. fallacy : logic
D. archetype : paradigm
E. debauchery : appetites
19
Ans :C

46. DETRITUS : GLACIERS


A. thaw : cold
B. snow : icebergs
C. sediment : bottom
D. silt : rivers
E. dregs : society

Ans :D

47. OUTSKIRTS : TOWN


A. water : goblet
B. margin : page
C. rung : ladder
D. hangar : airplane
E. trunk : tree

Ans :B

48. EQUIVOCATE : COMMITMENT


A. collaborate : falsification
B. fabricate : explanation
C. procrastinate : action
D. expostulate : confusion
E. implicate : exposition

Ans :C

49. MORPHINE : SEDATES


A. oil : smears
B. bandage : protects
C. drug : addicts
D. liquor : sedates
E. medicine : soothes

Ans :D

50. STICKLER : APPROXIMATION


A. Lluggard : indolence
B. connoisseur : anachronism
C. scientist : theorizing
D. leader : guidance
E. purist : adulteration

Ans :E

51. LOOM : WEAVE


A. couch : sleep
B. needle : knit
C. soap : wash
D. machine : stitch
E. bail : fish

Ans :B

52. SUBPOENA : WITNESS


A. hire : laborer
B. tax : worker
20
C. elect : officer
D. suborn : judge
E. conscript : soldier

Ans :E

53. INVINCIBLE : SUBDUED


A. expensive : bought
B. inconsistent : expressed
C. bolted : separated
D. impervious : damaged
E. imprudent : enacted

Ans :D

54. BURLESQUE : PLAY


A. operetta : symphony
B. limerick : sonnet
C. doggerel : verse
D. table : narration
E. sketch : drawing

Ans :C

55. GROW : BURGEON


A. flourish : thrive
B. transport : enrapture
C. beat : palpitate
D. evolve : multiply
E. wrot : decay

Ans :C

56. HOAX : DECEIVE


A. gimmick : wheedle
B. filibuster : delay
C. boast : cajole
D. lottery : disburse
E. scandal : vilify

Ans :B

57. BODY GUARD : PERSON


A. teacher : pupil
B. mayor : city
C. police officer : traffic
D. soldier : country
E. secretary : office

Ans :D

58. MUFFLE : SOUND


A. conceal : secret
B. assuage : grief
C. maul : object
D. extract : flavor
E. endure : agony

Ans :B
21
59. CENSORSHIP : INFORMATION
A. cultivation : erosion
B. philanthropy : generosity
C. frugality : constraint
D. sampling : measurement
E. sanitation : disease

Ans :E

60. DUPLICATE : ALTER


A. greet : ignore
B. exchange : return
C. shake : stabilize
D. stretch : shrink
E. eradicate : implicate

Ans :D

61. HYPOTHESIS : EXPERIMENTATION


A. reality : fantasy
B. opinion : debate
C. film : camera
D. predication : conclusion
E. science : success

Ans :B

62. TANGO : DANCE


A. stanza : line
B. tonality : instrumentation
C. arabesque : theme
D. rhyme : pattern
E. elegy : poem

Ans :E

63. CHRONOMETER : SUNDIAL


A. measurement : visibility
B. chronology : analogy
C. computer : abacus
D. watch : ray
E. reduction : enlargement

Ans :C

64. FOOTBALL : GRIDIRON


A. wrestling : mat
B. court : tennis
C. bowling : floor
D. rugby : arena
E. baseball : diamond

Ans :E

65. EXPURGATE : PASSAGE


A. abridge : text
B. filter : water
C. irritate : wound
D. burn : book
E. cancel : plan
22
Ans :B

66. IMPECUNIOUS : HOVEL


A. progress : prosper
B. mendicant : evasion
C. prosperity : poverty
D. mendacious : cringe
E. affluent : mansion

Ans :E

67. APIARY : BEE


A. museum : painting
B. dam : water
C. arboretum : tree
D. forum : speech
E. planetarium : star

Ans :C

68. BULLET : BARREL


A. fame : films
B. train : track
C. idea : brain
D. plane : clouds
E. water : boat

Ans :B

69. VINDICATE : REPREHENSIBLE.


A. sad : sorrow
B. bitter : sad
C. mild : serious
D. solid : porous
E. vivid : obsequious.

Ans :C

70. TERMAGANT : SHREW


A. virago : harpy
B. anteater : mouse
C. supporter : nag
D. single : married
E. male : female.

Ans :A

71. APOSTATE : RELIGION


A. traitor : country
B. renegade : Indian
C. loyal : faith
D. vital : church
E. diloyal : colonies.

Ans :A

72. PLEBISCITE : UKASE


A. lack : abundance
B. vote : musical instrument
23
C. cancel : construct
D. public : ruler
E. written : oral

Ans :D

73. DEBATER : LARYNGITIS


A. pedestrian : lameness
B. actor : aplause
C. doctor : diagnosis
D. swimmer : wet
E. writer : paper

Ans :A

74. INKBLOT : EYECHART


A. oculist : ophthalmologist
B. blotter : spectacles
C. psychiatrist : optometrist
D. physician : specialist
E. blurs : letters

Ans :C

75. LIGNEOUS : WOOD


A. cellular : microbe
B. nautical : water
C. igneous : rock
D. osseous : bone
E. fossilized : plant

Ans :D

76. SHRINE : PILGRIM


A. defeat : loser
B. peak : climber
C. rescue : danger
D. election : contestant
E. direction : driver.

Ans :B

77. RIVAL : COMPETITION


A. litigant : morality
B. maverick : co-operation
C. mentor : praise
D. sycophant : flattery
E. medicant : confusion.

Ans :D

78. SPIKE : TACK


A. bullet : wound
B. knife : cut
C. arrow : bow
D. spear : dart
E. pin : needle

Ans :D
24
79. INIQUITOUS : DISOBEDIENT
A. adult : child
B. hostile : cool
C. quiescent : lethargic
D. inflammable : flammable
E. inequitable : equitable

Ans :B

80. BALEFUL : MENACE


A. brusque : retort
B. competent : achievement
C. placid : boredom
D. flirtatious : affection
E. solicitous : concern

Ans :E

81. ALACRITY : APATHETIC


A. compliance : deft
B. temerity : timid
C. despotism : arrogant
D. candor : bungling
E. tenacity : eager

Ans :B

82. FIRE : STORM


A. whale : minnow
B. speech : shout
C. plant : flower
D. wind : temperature
E. tornado : hurricane

Ans :E

83. COLLUSION : CONSPIRATORS


A. identification : arbitrators
B. co-operation : partners
C. conclusion : messengers
D. revision : corespondents
E. attribution : interpreters

Ans :B

84. LIQUEFY : PETRIFY


A. cash in : strengthen
B. insolvent : bankrupt
C. water : stone
D. soften : frighten
E. solvent : rich

Ans :C

85. AMBULATORY : BEDRIDDEN


A. strong : weak
B. wheelchair : bed
C. free : confined
D. healthy : sick
E. broken : arm
25
Ans :C

86. CYNOSURE : BRILLIANT


A. word : common
B. student : attentive
C. rock : large
D. magnet : attractive
E. map : legible

Ans :D

Directions:
Each reading passage in this section is followed by questions based on the content of the reading passage. Read the
passage carefully and chose the best answer to each question. The questions are to be answered on the basis of
what is stated or implied in the passage.

1. But man is not destined to vanish. He can be killed, but he cannot be destroyed, because his soul is
deathless and his spirit is irrepressible. Therefore, though the situation seems dark in the context of the
confrontation between the superpowers, the silver lining is provided by amazing phenomenon that the very
nations which have spent incalculable resources and energy for the production of deadly weapons are
desperately trying to find out how they might never be used. They threaten each other, intimidate each other
and go to the brink, but before the total hour arrives they withdraw from the brink.
1. The main point from the author's view is that
A. Man's soul and spirit can not be destroyed by superpowers.
B. Man's destiny is not fully clear or visible.
C. Man's soul and spirit are immortal.
D. Man's safety is assured by the delicate balance of power in terms of nuclear weapons.
E. Human society will survive despite the serious threat of total annihilation.

Ans : E

2. The phrase 'Go to the brink' in the passage means


A. Retreating from extreme danger.
B. Declare war on each other.
C. Advancing to the stage of war but not engaging in it.
D. Negotiate for peace.
E. Commit suicide.

Ans : C

3. In the author's opinion


A. Huge stockpiles of destructive weapons have so far saved mankind from a catastrophe.
B. Superpowers have at last realized the need for abandoning the production of lethal
weapons.
C. Mankind is heading towards complete destruction.
D. Nations in possession of huge stockpiles of lethal weapons are trying hard to avoid actual
conflict.
E. There is a Silverlining over the production of deadly weapons.

Ans : D

4. 'Irrepressible' in the second line means


A. incompatible
B. strong
C. oppressive
D. unrestrainable
E. unspirited

Ans : D
26
5. A suitable title for the above passage is
A. Destruction of mankind is in evitable.
B. Man's desire to survive inhibits use of deadly weapons.
C. Mounting cost of modern weapons.
D. Threats and intimidation between super powers.
E. Cowardly retreat by man

Ans : B

2. Disequilibrium at the interface of water and air is a factor on which the transfer of heat and water vapor from
the ocean to the air depends. The air within about a millimeter of the water is almost saturated with water
vapor and the temperature of the air is close to that of the surface water. Irrespective of how small these
differences might be, they are crucial, and the disequilibrium is maintained by air near the surface mixing
with air higher up, which is typically appreciably cooler and lower in water vapor content. The turbulence,
which takes its energy from the wind mixes the air. As the speed of wind increases, so does the turbulence,
and consequently the rate of heat and moisture transfer. We can arrive at a detailed understanding of this
phenomenon after further study. The transfer of momentum from wind to water, which occurs when waves
are formed is an interacting-and complicated phenomenon. When waves are made by the wind, it transfers
important amounts of energy-energy, which is consequently not available for the production of turbulence.
1. This passage principally intends to:
A. resolve a controversy
B. attempt a description of a phenomenon
C. sketch a theory
D. reinforce certain research findings
E. tabulate various observations

Ans : B

2. The wind over the ocean usually does which of the following according to the given
passage?
I. Leads to cool, dry air coming in proximity with the ocean surface.
II. Maintains a steady rate of heat and moisture transfer between the ocean and the air.
III. Results in frequent changes in the ocean surface temperature.
A. I only
B. II only
C. I and II only
D. II and III only
E. I, II, and III

Ans : A

3. According to the author the present knowledge regarding heat and moisture transfer from
the ocean to air as
A. revolutionary
B. inconsequential
C. outdated
D. derivative
E. incomplete

Ans : E

4. According to the given passage, in case the wind was to decrease until there was no wind at
all, which of the following would occur?
A. The air, which is closest to the ocean surface would get saturated with water vapor.
B. The water would be cooler than the air closest to the ocean surface.
C. There would be a decrease in the amount of moisture in the air closest to the ocean
surface.
D. There would be an increase in the rate of heat and moisture transfer.
E. The temperature of the air closest to the ocean and that of the air higher up would be the
same.
27
Ans : A

3. The Food and Drug Administration has formulated certain severe restrictions regarding the use of
antibiotics, which are used to promote the health and growth of meat animals. Though the different types of
medicines mixed with the fodder of the animals kills many microorganisms, it also encourages the
appearance of bacterial strains, which are resistant to anti-infective drugs.

It has already been observed that penicillin and the tetracyclines are not as effective therapeutically as they
once used to be. This resistance to drugs is chiefly caused due to tiny circlets of genes, called plasmids,
which are transferable between different species of bacteria. These plasmids are also one of the two kinds
of vehicles on which molecular biologists depend on while performing gene transplant experiments. Existing
guidelines also forbid the use of plasmids, which bear genes for resistance to antibiotics, in the laboratories.
Though congressional dabate goes on as to whether these restrictions need to be toughened with reference
to scientists in their laboratories, almost no congressional attention is being paid to an ill advised agricultural
practice, which produces deleterious effects.

1. In the present passage, the author's primary concern is with:


A. The discovery of methods, which eliminate harmful microorganisms without generating
drug-resistant bacteria.
B. Attempting an explanation of the reasons for congressional inaction about the regulation
of gene transplant experiments.
C. Portraying a problematic agricultural practice and its serious genetic consequences
D. The verification of the therapeutic ineffectiveness of anti-infective drugs
E. Evaluation of the recently proposed restrictions, which are intended to promote the growth
of meat animals.

Ans : C

2. As inferred from the above passage, the mutual transfer of plasmids between different
bacteria can result in which of the following?
A. Microorganisms, which have an in-built resistance to drugs
B. Therapeutically useful circlets of genes
C. Penicillin like anti-infective drugs
D. Viruses used by molecular biologists
E. Carriers for performing gene transplant experiments.

Ans : A

3. According to the above passage the author believes that those who favor the stiffening of
restrictions on gene transplant research should logically also.
A. Approve and aid experiments with any plasmids except those, which bear genes for
antibiotic resistance.
B. Inquire regarding the addition of anti-infective drugs to livestock feeds
C. Oppose the using of penicillin and tetracyclines in order to kill microorganisms
D. Agree to the development of meatier live-stock through the use of antibiotics
E. Approve of congressional debate and discussion regarding science and health issues.

Ans : B

4. The attitude the author has with reference to the development of bacterial strains that
render antibiotic drugs in effective can best be described as
A. indifferent
B. perplexed
C. pretentious
D. insincere
E. apprehensive

Ans : E
28
4. Roger Rosenblatt's book Black Fiction, manages to alter the approach taken in many previous studies by
making an attempt to apply literary rather than sociopolitical criteria to its subject. Rosenblatt points out that
criticism of Black writing has very often served as a pretext for an expounding on Black history. The recent
work of Addison Gayle's passes a judgement on the value of Black fiction by clearly political standards,
rating each work according to the ideas of Black identity, which it propounds.

Though fiction results from political circumstances, its author react not in ideological ways to those
circumstances, and talking about novels and stories primarily as instruments of ideology circumvents much
of the fictional enterprise. Affinities and connections are revealed in the works of Black fiction in Rosenblatt's
literary analysis; these affinities and connections have been overlooked and ignored by solely political
studies.

The writing of acceptable criticism of Black fiction, however, presumes giving satisfactory answers to a quite
a few questions. The most important of all, is there a sufficient reason, apart from the racial identity of the
authors, for the grouping together of Black authors? Secondly, what is the distinction of Black fiction from
other modern fiction with which it is largely contemporaneous? In the work Rosenblatt demonstrates that
Black fiction is a distinct body of writing, which has an identifiable, coherent literary tradition. He highlights
recurring concerns and designs, which are independent of chronology in Black fiction written over the past
eighty years. These concerns and designs are thematic, and they come form the central fact of the
predominant white culture, where the Black characters in the novel are situated irrespective of whether they
attempt to conform to that culture or they rebel against it.

Rosenblatt's work does leave certain aesthetic questions open. His thematic analysis allows considerable
objectivity; he even clearly states that he does not intend to judge the merit of the various works yet his
reluctance seems misplaced, especially since an attempt to appraise might have led to interesting results.
For example, certain novels have an appearance of structural diffusion. Is this a defeat, or are the authors
working out of, or attempting to forge, a different kind of aesthetic? Apart from this, the style of certain Black
novels, like Jean Toomer's Cane, verges on expressionism or surrealism; does this technique provide a
counterpoint to the prevalent theme that portrays the fate against which Black heroes are pitted, a theme
usually conveyed by more naturalistic modes of expressions?

Irrespective of such omissions, what Rosenblatt talks about in his work makes for an astute and worthwhile
study. His book very effectively surveys a variety of novels, highlighting certain fascinating and little-known
works like James Weldon Johnson's Autobiography of an Ex-Coloured Man. Black Fiction is tightly
constructed, and levelheaded and penetrating criticism is exemplified in its forthright and lucid style.

1. The author of the passage raises and objection to criticism of Black fiction like that by
Addison Gayle as it:
A. Highlights only the purely literary aspects of such works
B. Misconceive the ideological content of such fiction
C. Miscalculate the notions of Black identity presented in such fiction
D. Replaces political for literary criteria in evaluating such fiction
E. Disregards the reciprocation between Black history and Black identity exhibited in such
fiction.

Ans : D

2. The primary concern of the author in the above passage is:


A. Reviewing the validity of a work of criticism
B. Comparing various critical approaches to a subject
C. Talking of the limitations of a particular kind of criticism
D. Recapitulation of the major points in a work of criticism
E. Illustrating the theoretical background of a certain kind of criticism.

Ans : A

3. The author is of the opinion that Black Fiction would have been improved had Rosenblatt:
A. Undertaken a more careful evaluation of the ideological and historical aspects of Black
Fiction
B. Been more objective in his approach to novels and stories by Black authors
C. Attempted a more detailed exploration of the recurring themes in Black fiction throughout
its history
29
D. Established a basis for placing Black fiction within its own unique literary tradition
E. Calculated the relative literary merit of the novels he analyzed thematically.

Ans : E

4. Rosenblatt's discussion of Black Fiction is :


A. Pedantic and contentious
B. Critical but admiring
C. Ironic and deprecating
D. Argumentative but unfocused
E. Stilted and insincere.

Ans : B

5. According to the given passage the author would be LEAST likely to approve of which
among the following?
A. Analyzing the influence of political events on the personal ideology of Black writers
B. Attempting a critical study, which applies sociopolitical criteria to the autobiographies of
Black authors
C. A literary study of Black poetry that appraises the merits of poems according to the
political acceptability of their themes
D. Studying the growth of a distinct Black literary tradition within the context of Black history
E. Undertaking a literary study, which attempts to isolate aesthetic qualities unique to Black
fiction.

Ans : C

6. From the following options, which does the author not make use of while discussing Black
Fiction?
A. Rhetorical questions
B. Specific examples
C. Comparison and contrast
D. Definition of terms
E. Personal opinion.

Ans : D

7. The author makes a reference to James Weldon Johnson's Autobiography of an Ex-colored


Man most probably to:
A. Highlight the affinities between Rosenblatt's method of thematic analysis and earlier
criticism
B. Elucidate regarding the point made regarding expressionistic style earlier in the passage
C. Qualify the assessment of Rosenblatt's book made in the first paragraph of the passage
D. Demonstrate the affinities among the various Black novels talked of by Rosenblatt's
literary analysis
E. Present a specific example of one of the accomplishments of Rosenblatt's work.

Ans : E

5. Some modern anthropologists hold that biological evolution has shaped not only human morphology but
also human behavior. The role those anthropologists ascribe to evolution is not of dictating the details of
human behavior but one of imposing constraints - ways of feeling, thinking, and acting that ''come naturally''
in archetypal situations in any culture. Our ''frailties'' - emotions and motivs such as rage, fear, greed,
gluttony, joy,lust, love-may be a very mixed assortment quality: we are, as we say, ''in the grip'' of them. And
thus they give us oursense of constraints.

Unhappily, some of those frailties our need for ever-increasing security among them are presently
maladaptive. Yet beneath the overlay of cultural detail, they, too, are said to be biological in direction, and
therefore as natural to us as are our appendixes. We would need to comprehend throughly their adaptive
origins in order to understand how badly they guide us now. And we might then begin to resist their
pressure.
30
1. The author implies that control to any extent over the ''frailties'' that constrain our behavior
is though to presuppose
A. That those frailties and adaptive are recognized as currently beneficial and adaptive
B. That there is little or no overlay of cultural detail that masks their true nature.
C. That there are cultures in which those frailties do not ''come naturally'' and from which
such control can be learned
D. A full understanding of why those frailties evolved and of how they function now
E. A thorough grasp of the principle that cultural detail in human behavior can differ arbitrarily
from society to society.

Ans : D

2. It can be inferred that in his discussion of maladaptive frailties the author assumes that
A. Evolution does not favor the emergence of adaptive characteristics over the emergence of
maladaptive ones
B. Any structure or behavior not positively adaptive is regarded as transitory in evolutionary
theory
C. Maladaptive characteristics, once fixed, make the emergence of other maladaptive
characteristics more likely
D. The designation of a characteristic as being maladaptive must always remain highly
tentative
E. Changes in the total human environment can outpace evolutionary change.

Ans : E

3. The primary purpose of the passage is to present


A. A position on the foundations of human behavior and on what those foundations imply
B. A theory outlining the parallel development of human morphology and of human behavior
C. A diagnostic test for separating biologically determined behavior patters from culture -
specific detail
D. An overview of those human emotions and motive's that impose constraints on human
behaviour
E. A practical method for resting the pressures of biologically determined drives.

Ans : A

4. Which of the following most probably provides an appropriate analogy from human
morphology for the ''details'' versus ''constraints'' distinction made in the passage in
relation to human behaviour?
A. The ability of most people to see all the colors of the visible spectrum as against most
peoples inability to name any but the primary colors
B. The ability of even the least fortunate people to show compassion as against people's
inability to mask their feelings completely
C. The ability of some people to dive to great depths as against most people's inability to
swim long distance
D. The psychological profile of those people who are able to delay gratification as against
people's inability to control their lives completely
E. The greater lung capacity of mountain peoples that helps them live in oxygen-poor air as
against people's inability to fly without special apparatus.

Ans : E

6. The existence of mammals on the earth can be traced back to at least the Triassic time. The rate of
development was retarded, till evolutional change suddenly accelerated in the oldest Paleocene. This
resulted in an increase in average size, larger mental capacity, and special adaptations for different modes
of life, during the Eocene time. Further improvement was seen during the Oligocene Epoch, with the
appearance of some new lines and extinction of others. The Miocene and Pliocene times are especially
significant as they mark the culmination of various groups and a continued approach toward modern
characters. It is in the Miocene time that the mammals reached their peak with reference to variety and size.
31
The ability of the mammals to adapt to various modes of life finds a parallel in the reptiles of the Mesozoic
time, and apart form their greater intelligence, the mammals apparently have not done much better than the
corresponding reptilian forms. Undoubtedly the bat is a better flying animal than the pterosaur, but at the
same time the dolphin and whale are hardly more fish like than the ichthyosaur. Quite a few of the swift-
running mammals inhabiting the plains, like the horse and the antelope, must excel any of the dinosaurs.
Although the tyrannosaur was a more weighty and robust carnivore than perhaps any carnivorous mammal,
the lion and the tiger, by virtue of their superior brain are far more efficient and dangerous beasts of prey. It
is significant to note that various species of mammals gradually adapted themselves to various kinds of
lifestyles, some took to grazing on the plains and were able to run swiftly (horse, deer, bison), others started
living in rivers and swamps (hippopotamus, beaver), inhabiting trees (sloth, monkey), burrowing
underground (rodent, mole), feeding on flesh (tiger, wolf), swimming in the water (dolphin, whale, seal), and
flying in the air (bat). Human beings on account of their superior brain have been able to harness
mechanical methods to conquer the physical world and adapt to any set of conditions.

Such adaptation to different conditions leads to a gradual change in form and structure. This is a biological
characteristic of the youthful, plastic stage of a group. It is seen that early in its evolutional cycle animals
possess the capacity for change, but as the animal progresses in its cycle becoming old and fixed, this
capacity for change disappears. The generalized types of organisms retain longest the ability to make
adjustments when required, and it is from them that new, fecund stocks take origin-certainly not from any
specialized end products. With reference to mammals, we see their birth, plastic spread in many directions,
increased specialization, and in some cases, extinction; this is a characteristic of the evolution of life, which
can be seen in the geologic record of life.

1. From the following, choose the most appropriate title for the above passage?
A. From Dinosaur to Man
B. Adaptation and Extinction
C. The Superior Mammals
D. The Geologic Life Span
E. Man, the Vanquisher of the Physical World.

Ans : B

2. According to the passage the chronological order of the geologic periods is:
A. Paleocene, Miocene, Triassic, Mesozoic
B. Paleocene, Triassic, Mesozoic, Miocene
C. Miocene, Paleocene, Triassic, Mesozoic
D. Mesozoic, Oligocene, Paleocene, Miocene
E. Mesozoic, Paleocene, Eocene, Miocene

Ans : E

3. From the above passage, we can infer that, the pterosaur


A. resembled the bat
B. was a Mesozoic mammal
C. was a flying reptile
D. inhabited the seas
E. evolved during the Miocene period

Ans : C

4. As inferred from the passage, the largest number of mammals were found in which of the
following periods?
A. Triassic period
B. Eocene period
C. Oligocene epoch
D. Pliocene period
E. Miocene period

Ans : E

5. Among the following statements, which statement, if true, would weaken the argument put
forth in the first sentence of Paragraph 1?
32
A. It has been found that the tryannosaur had a larger brain, than was previously known.
B. Within the next thousand years, mammals will become extinct.
C. Recently certain forms of flying ichthyosaurs have been discovered.
D. It has now been proved, that the tiger is more powerful than the carnivorous reptiles.
E. It is now possible to double human mental capacity, by the use of certain recently
developed computers.

Ans : A

6. It is clear from the passage, that the evidence used to discuss the life of past time periods
A. was developed by Charles Darwin
B. was unearthed by the author
C. has been negated by more recent evidence
D. was never truly established
E. is based on fossilized remains

Ans : E

7. As inferred from the passage, which of the following proverbial expressions is the author
most likely to agree with?
A. It's a cruel world.
B. All the world's a stage.
C. The more things change, the more they remain the same.
D. Footprints in the sands of time.
E. A short life, but a merry one.

Ans : D

7. For a period of more than two centuries paleontologists have been intrigued by the fossilized remains of
pterosaurs, the first flying vertebartes. The issues, which puzzle them, are how these heavy creatures,
having a wingspan of about 8-12 meters managed the various problems associated with powered flight and
whether these creatures were reptiles or birds.

Perhaps the least controversial assertion about the pterosaurs is that they were reptiles. Their skulls,
pelvises, and hind feet are reptilian. The anatomy of their wings suggests that they did not evolve into the
class of birds. In pterosaurs a greatly elongated fourth finger of each forelimb supported a winglike
membrane. The other fingers were short and reptilian, with sharp claws. In birds the second finger is the
principal strut of the wing, which consists primarily of feathers. If the pterosaurs walked on all fours, the three
short fingers may have been employed for grasping. When a pterosaurs walked or remained stationary, the
fourth finger, and with it the wing, could only urn upward in an extended inverted V- shape along each side
of the animal's body.

In resemblance they were extremely similar to both birds and bats, with regard to their overall body structure
and proportion. This is hardly surprising as the design of any flying vertebrate is subject to aerodynamic
constraints. Both the pterosaurs and the birds have hollow bones, a feature that represents a savings in
weight. There is a difference, which is that the bones of the birds are more massively reinforced by internal
struts.

Although scales typically cover reptiles, the pterosaurs probably had hairy coats. T.H. Huxley reasoned that
flying vertebrates must have been warm-blooded because flying implies a high rate of metabolism, which in
turn implies a high internal temperature. Huxley speculated that a coat of hair would insulate against loss of
body heat and might streamline the body to reduce drag in flight. The recent discovery of a pterosaur
specimen covered in long, dense, and relatively thick hair like fossil material was the first clear evidence that
his reasoning was correct.

Some paleontologists are of the opinion that the pterosaurs jumped from s dropped from trees or perhaps
rose into the light winds from the crests of waves in order to become airborne. Each theory has its
associated difficulties. The first makes a wrong assumption that the pterosaurs hind feet resembled a bat's
and could serve as hooks by which the animal could hang in preparation for flight. The second hypothesis
seems unlikely because large pterosaurs could not have landed in trees without damaging their wings. The
third calls for high aces to channel updrafts. The pterosaurs would have been unable to control their flight
once airborne as the wind from which such waves arose would have been too strong.
33
1. As seen in the above passage scientists generally agree that:
A. the pterosaurs could fly over large distances because of their large wingspan.
B. a close evolutionary relationship can be seen between the pterosaurs and bats, when the
structure of their skeletons is studied.
C. the study of the fossilized remains of the pterosaurs reveals how they solved the problem
associated with powered flight
D. the pterosaurs were reptiles
E. Pterosaurs walked on all fours.

Ans : D

2. The view that, the pterosaurs rose into light winds from the crest of the waves to become
airborne, is viewed by the author as
A. revolutionary
B. unlikely
C. unassailable
D. probable
E. outdated.

Ans : B

3. As inferred from the passage, the skeleton of a pterosaur is distinguishable from that of a
bird by the
A. length of its wingspan
B. hollow spaces in its bones
C. anatomic origin of its wing strut
D. evidence of the hooklike projections on its hind feet
E. location of the shoulder joint joining the wing to its body.

Ans : C

4. From the viewpoint of T.H.Huxley, as given in the passage, which of the following
statements is he most likely to agree with?
A. An animal can master complex behaviors irrespective of the size of it's brain.
B. Environmental capabilities and physical capabilities often influence the appearance of an
animal.
C. Usually animals in a particular family group do not change their appearance dramatically
over a period of time
D. The origin of flight in vertebrates was an accidental development rather than the outcome
of specialization or adaption
E. The pterosaurs should be classified as birds, not reptiles.

Ans : B

5. According to the passage which of the following is a characteristic of the pterosaurs?


A. The pterosaurs were not able to fold their wings when not in use
B. Like the bats, they hung upside down from branches
C. They flew in order to capture prey
D. They can be said to be an earlier stage in the evolution of the birds
E. They lived principally in a forest like habitat.

Ans : A

6. The organization of the last paragraph of the passage can best be described as:
A. New data is introduced in order to support a traditional point of view
B. Three explanations are put forth and each of them is disputed by means of specific
information
C. An outline of three hypotheses are given and evidence supporting each of them is given
D. Description of three recent discoveries is presented, and their implications for future study
are projected
E. The material in the earlier paragraphs is summarized and certain conclusions are from it.
34
Ans : B

7. According to the passage, some scientists believe that pterosaurs


A. Lived near large bodies of water
B. Had sharp teeth for tearing food
C. Were attacked and eaten by larger reptiles
D. Had longer tails than many birds
E. Consumed twice their weight daily to maintain their body temperature.

Ans : A

8. Certain scraps of evidence bear out those who hold a very high opinion of the average level of culture
among the Athenians of the great age. Pericles's funeral speech is undoubtedly the most famous evidence
from Athenian literature, that its level was indeed high. However, Pericles was a politician, and it is possible
that he was flattering his audience. We know that thousands of Athenians sat hour after hour in the theater
listening to the plays of the great Greek dramatists. The Greek plays, particularly the tragedies, maintained
an extremely high intellectual level throughout, with no letdowns, no concessions to the lowbrows or to the
demands of ''realism'', like the gravediggers scene in Shakespeare's Hamlet. The music and dancing seen in
these plays were also of an equally high level. The best modern parallel can be seen in the restrained,
difficult opera of the 18th century. The comparison is no doubt dangerous, but can you imagine almost the
entire population of an American city (in suitable installments, of course) sitting through performances of
Mozart's Don Giovanni or Gluck's Orpheus? Perhaps the Athenian masses went to these plays because of a
lack of other amusements. They could at least understand something of what went on, since the subjects
were part of their folklore. Undoubtedly the theme of grand opera is not part of the folklore of the American
people.
1. From the passage it is evident that the author seems to question the sincerity of
A. politicians
B. playwrights
C. opera goers
D. ''low brows''
E. gravediggers.

Ans : A

2. According to the author the average American


A. Enjoys Hamlet
B. Loves folklore
C. Is not able to understand grand opera
D. Seeks a high cultural level
E. Lacks entertainment.

Ans : C

3. From the passage, we can say that the author's attitude toward Greek plays is one of
A. Qualified approval
B. Grudging admiration
C. Studied indifference
D. Partial hostility
E. Great respect.

Ans : E

4. The author makes a suggestion that Greek plays


A. Were demanding on the actors
B. Flattered their audiences
C. Were focussed on a limited audience
D. Were dominated by music and dancing
E. Stimulated their audiences.

Ans : E
35
9. Everyone conforms to infancy, infancy conforms to nobody, so that one babe commonly makes four or five
out of the adults who prattle and play to it. So God has armed youth and puberty and manhood no less with
its own piquancy and charm, and made it enviable and gracious and its claims not to be put by, if it will stand
by itself. Do not think the youth has no force, because he cannot speak to you and me. Hark! In the next
room his voice is sufficiently clear and emphatic. It seems he knows how to speak to his contemporaries.
Bashful or bold, then, he will know how to make us seniors very unnecessary.

The healthy attitude of human nature can be seen in the nonchalance of boys who are sure of a dinner, and
would disdain as much as a lord to do or say aught to conciliate one. A boy is in the parlor what the pit is in
the playhouse; independent, irresponsible, looking out from his corner on such people and facts as pass by,
he tries and sentences them on their merits, in the swift, summary way of boys, as good, bad, interesting,
silly, eloquent, troublesome. He never cumbers himself regarding consequences, about interests and he
gives an independent, genuine verdict. You should court him: he will not court you. But the man is, as it
were, clapped into jail by his consciousness. As soon as he has once acted or spoken with eclat, he is a
committed person, watched by the sympathy or the hatred of hundreds, whose affections must now enter
into his account. There is no Lethe for this. Ah, that he could pass again into his neutrality.

These are the voices, which we hear in solitude, but they grow faint and inaudible as we enter into the world.
Everywhere society is conspiring against the manhood of every one of its members. Society is joint - stock
company, in which members agree, for the better securing of his bread to each shareholder, to surrender
the liberty and culture of the eater. The virtue in most request is conformity. It is averse to self-reliance. What
it loves is names and customs and not realities and creators.

Whosoever is a man has to be a nonconformist. He who would gather immortal palms must not be hindered
by the name of goodness, but must explore if it be goodness. Nothing is at last sacred but the integrity of
your own mind.

No law can be sacred to me but that of my nature. Good and bad are but names very readily transferable to
that to this; the only right is what is after my constitution, the only right is what is after me constitution, the
only wrong what is against it. A man is to carry himself in the presence of all opposition as if every thing
were titular and ephemeral but he. I am ashamed to think how easily we capitulate to badges and names, to
large societies and dead institutions. Every decent and well-spoken individual affects and sways me more
than is right. I ought to go upright and vital, and speak the rude truth in all ways.

I shun father and mother and wife and brother, when my genius calls me. I would write on the lintels of the
doorpost, whim. I hope it is somewhat better than whim at last, but we cannot spend the day in explanation.
Except me not to show cause why I seek or why I exclude company. Then, again, do not tell me, as a good
man did not to-day, of my obligation to put all poor men in good situations. Are they my poor? I tell thee,
thou foolish philanthropist, that I grudge the dollar, the time, the cent, I give to such men as do not belong to
me and to whom I do not belong. There is a class of person to whom by all spiritual affinity I am bought and
sold; for them I will go to prison, if need be; but your miscellaneous popular charities; the education at
collage of fools; the building of meeting - house to the vain end to which many now stand; alms to sots; and
the thousandfold Relief Societies; - though I confess with shame I sometimes succumb and give the dollar, it
is a wicked dollar which by and by I shall have the manhood to withhold.

If you refuse to conform, you can experience the displeasure of the world. Hence, a man should know how
to estimate a sour face. The by - standers look askance on him in the public street or in the friend's parlor. In
case this aversion originates from contempt and resistance similar to his own, it might result in a sad
countenance; but the sour faces of the multitude, like their sweet faces, have no deep cause, but are caused
by reasons as diverse as the direction of the wind and what he reads in the newspapers. Yet is the
discontent of the multitude more formidable than that of the senate and the collage.

Another factor, which frightens us from self - trust in our consistency; a reverence for our past act or word,
because the eyes of others have no other data for computing our orbit than our past acts, and we are loath
to disappoint them.

But why should you keep your head over your shoulder? Why drag about this corpse of your memory, lest
you contradict somewhat you have stated in this or that public place? Suppose you should contradict
yourself; what then?

This is a rather silly consistency in our minds, which is adored by little statesmen and philosophers and
divines. Uniformly a great soul has almost nothing to do, he could just occupy himself with his shadow on
the wall. Speak what you think now in hard words; and to-morrow speak what tomorrow thinks in hard words
36
again, though it contradict everything you said to-day. - ''Ah, so you shall be sure to be misunderstood.'' - Is
it so bad, then, to be misunderstood? Pythagoras was misunderstood, and Socrates, and Jesus, and Luther,
and Copernicus, and Galileo, and Newton, and every pure and wise spirit that ever took flesh. What can be
considered to be truly great is to be misunderstood.

1. Which of the following statements would best describe the main theme of the above
passage?
A. "A foolish consistency is the hobgoblin of little mind."
B. "Eternal youth means eternal independence."
C. "Whoso would be a man must be a nonconformist."
D. "Colleges are designed to educate fools."
E. "Infancy conforms to nobody."

Ans : C

2. When is the period during which we are most nonconformist?


A. infancy
B. puberty
C. youth
D. manhood
E. old age

Ans : A

3. In his statement ''What can be considered to be truly great is to be misunderstood'' the


author means:
A. One should refrain from saying, what one exactly means
B. Being misunderstood, equals being great
C. All great man have always been misunderstood
D. Even though a person might be considered inconsistent, he shouldn't hesitate to change
his mind if he feels the need to.
E. It is seldom, that nice people succeed

Ans : D

4. As inferred from the passage, the refusal of young people to cater to accept public opinion
is:
A. A feature of the rebelliousness of youth
B. A healthy attitude of human nature
C. A manifestation of deep - seated immaturity
D. Simply bad manners
E. Part of growing up

Ans : B

5. "Society is a joint-stock company etc." is one way which the author shows
A. The anti-culture attitude of the public
B. Society is highly organized and structured
C. The self-rejection of society
D. The lack of room for solitude in our world
E. The public's interest in the stock market

Ans : C

6. " I would write on the lintels of the doorpost, whim." What does the author mean by this
statement:
A. That one should renounce his immediate family
B. That signposts have an important educational function in our society
C. That an impulsive action may have a subsequent rational explanation
D. That one must never be held responsible for what one says and does
E. That everyone should do foolish things occasionally
37
Ans : C

7. Which of the following statements best summarizes the spirit and sense of the above
passage?
A. "Nothing is at last sacred but the integrity of your own mind."
B. "With consistency, a great soul; has simply nothing to do."
C. "Do not think the youth has no force, because cannot speak to you and me."
D. "The virtue in most request is conformity."
E. "A man must know how to estimate a sour force."

Ans : A

10. Furthermore, insofar as any conclusion about its author can be drawn from five or six plays attributed to him,
the Wakefield Master is without exception considered to be a man of sharp contemporary observation. He
was, probably clerically educated, as indicated by his Latin and music, his Biblical and patristic lore. Even
today he is remembered for his his quick sympathy for the oppressed and forgotten man, his sharp eye for
character, a ready ear for colloquial, vernacular turns of speech and a humor alternately rude and
boisterous, coarse and happy. Therefore in spite of his conscious artistry as can be seen in his feeling for
intricate metrical and stanza forms, he is regarded as a kind of medieval Steinbeck, indignantly angry at,
uncompromisingly and even brutally realistic in presenting the plight of the agricultural poor.

It is now fairly accepted to regard the play as a kind of ultimate point in the secularization of the medieval
drama. Therefore more stress has been laid on it as depicting realistically humble manners and pastoral life
in the bleak of the west riding of Yorkshire on a typically cold night of December 24th. After what are often
regarded as almost ''documentaries'' given in the three successive monologues of the three shepherds,
critics go on to affirm that the realism is then intensified into a burlesque mock-treatment of the Nativity.
Finally as a sort of epilogue or after-thought in deference to the Biblical origins of the materials, the play
slides back into an atavistic mood of early innocent reverence. In actuality, the final scene is the culminating
scene and also the raison d’etre of the introductory ''realism.''

Superficially the present play supports the conventional view of its mood of secular realism. At the same
time, the ''realism'' of the Wakefield Master is of a paradoxical turn. His wide knowledge of people, as well as
books indicates no cloistered contemplative but one in close relation to his times. Still, that life was after all a
predominantly religious one, a time which never neglected the belief that man was a rebellious and sinful
creature in need of redemption . So deeply (one can hardly say ''naively'' of so sophisticated a writer) and
implicitly religious is the Master that he is less able (or less willing) to present actual history realistically than
is the author of the Brome Abraham and Isaac. His historical sense is even less realistic than that of
Chaucer who just a few years before had done for his own time ''costume romances,'' such as The Knight's
Tele, Troilus and Cressida, etc. Furthermore, used highly romantic materials, which could excuse his taking
liberties with history.

1. Of the following statements, which is not true of Wakefield Master?


A. He and Chaucer were contemporaries.
B. Wakefield Master is remembered as having written five or six realistic plays.
C. His plays realistically portray the plight of the country folk of his day
D. His writing was similar to that of John Steinbeck.
E. He was an accomplished artist.

Ans : D

2. The word 'patristic' in the first paragraph is used to mean:


A. patriotic
B. superstitious
C. folk
D. relating to the Christian Fathers
E. realistic

Ans : D

3. The statement about the ''secularization of the medieval drama'' (opening sentence of the
second paragraph) refers to the
38
A. Introduction of religious themes in the early days
B. Presentation of erudite material
C. Use of contemporary materials
D. Return to early innocent reverence at the end of the play
E. Introduction of mundane matters in religious plays

Ans : E

4. From the following what would the writer be expected to do in the subsequent paragraphs:
A. Make a justification for his comparison with Steinbeck
B. Put forth a view point, which would take up the thought of the second paragraph
C. Point out the anachronisms in the play
D. Discuss the works of Chaucer
E. Talk about the lack of realism in the works of the Wakefield Master.

Ans : B

Verbal Section : Antonyms

Directions:
Each of the GRE sample antonyms questions consists of a word followed by five words or phrase as choices.
Choose the word or phrase which is most nearly opposite in meaning to the word in capitals and shade the alphabets
marked in the grid on your answer sheet.

Following are some GRE sample antonyms qustions.

1. ABOMINATE :
A. loathe
B. despise
C. adore
D. abhor
E. attach

Ans : C

2. OBSEQUIOUS :
A. servile
B. first
C. fawning
D. supercilious
E. improper

Ans : D

3. OROTUND :
A. not resonant
B. not reddish
C. not eager
D. pompous
E. loud

Ans : A

4. RECANT :
A. entangle
B. rescue
C. fail
D. assert
E. predict
39
Ans : D

5. UPBRAID :
A. defer
B. vacillate
C. sever
D. conjoin
E. laud

Ans : E

6. PLENITUDE :
A. luxury
B. magnificence
C. richness
D. contentment
E. scarcity

Ans : E

7. SCURRILOUS :
A. decent
B. savage
C. major
D. volatile
E. scabby

Ans : A

8. FULMINATION :
A. praise
B. repetition
C. escape
D. ratification
E. addition

Ans : A

9. DISTEND
A. deteriorate
B. weaken
C. constrict
D. concentrate
E. fold

Ans : C

10. TOUT
A. cast aspersions on
B. deny the relevance of
C. placate
D. withhold consent
E. misrepresent

Ans : E

11. SQUALID
A. fervid
B. florid
40
C. pristine
D. extraneous
E. abundant

Ans : C

12. SCOTCH
A. renovate
B. entrust
C. unfasten
D. encourage
E. emphasize

Ans : D

13. PERFIDY
A. tact
B. generosity
C. thoroughness
D. loyalty
E. gratitude

Ans : D

14. OUTLANDISH
A. conventional
B. prolific
C. unchanging
D. transparent
E. noticeable

Ans : A

15. PLUMB
A. reversed
B. lofty
C. horizontal
D. thin
E. light

Ans : C

16. FERVID
A. undistinguished
B. unexpected
C. stubborn
D. restrained
E. discouraged

Ans : D

17. VACUITY
A. quality
B. certainty
C. plenitude
D. stability
E. incontinence

Ans : C
41
18. RAVEL
A. knit
B. omit
C. remain silent
D. measure
E. increase in value

Ans : A

19. PERSISTENCE
A. irrelevance
B. inconstancy
C. inequality
D. intemperance
E. incompetence.

Ans : B

20. SUBROSA
A. openly
B. fashionably
C. under the owse
D. simply
E. clandestinely

Ans : A

21. INVIDIOUS :
A. candid
B. stubborn
C. defensive
D. hostile
E. inoffensive

Ans : E

22. MACERATE :
A. cover by painting
B. assess by observing
C. harden by drying
D. influence by lying
E. cure by medicating

Ans : B

23. SKEPTICISM :
A. reason
B. conviction
C. plausibility
D. audricty
E. argument

Ans : D

24. IGNOMINIOUS :
A. scholarly
B. incognito
C. laudatory
D. disgraceful
E. erudite
42
Ans : B

25. CODA :
A. creflain
B. crescendo
C. prelude
D. improvisation
E. solo

Ans : A

26. PALTRY :
A. farm
B. scanty
C. excessive
D. friendly
E. benevolent

Ans : A

27. PUISSANCE :
A. strength
B. knowledge
C. liberality
D. skepticism
E. powerlessness

Ans : E

28. MANUMIT :
A. print
B. impress
C. enslave
D. fail
E. endeavor

Ans : D

29. GENUFLECT :
A. pronounce correctly
B. falsify
C. trick
D. stand erect
E. project

Ans : E

30. INNOCUOUS :
A. toxic
B. large
C. sober
D. impeccable
E. spotless

Ans : C

31. ANIMOSITY
A. parody
B. retardation
C. sincerity
43
D. refutation
E. canaraderie

Ans : E

32. INVETERATE
A. uninvited
B. illiterate
C. cumulative
D. beginning
E. incompetent

Ans : D

33. SCOTCH
A. renovate
B. encourage
C. entrust
D. ameliorate

Ans : B

34. PREDILECTION
A. ambiguity
B. unwillingness to choose
C. desire to please
D. propensity to dislike
E. stereotype

Ans : D

35. CHOLERIC
A. good-natured
B. spoiled
C. irascible
D. immune
E. idiotic.

Ans : A

36. EXACERBATE
A. contemplate
B. bewilder
C. reward
D. better
E. horify

Ans : D

37. EQUANIMITY
A. clamour
B. volume
C. disparity
D. agitation
E. caution

Ans : D

38. ANIMADVERSION
44
A. gullibility
B. precision
C. praise
D. sobriety
E. criticize

Ans : C

39. EXHUME
A. enter
B. fertilize
C. inter
D. decay
E. clarify

Ans : C

40. CALLOW
A. rustic
B. crude
C. exquisite
D. experienced
E. migratory

Ans : D

41. CUPIDITY
A. generosity
B. love
C. anxiety
D. entertainment
E. tragedy.

Ans : A

42. ANIMOSITY
A. parody
B. retardation
C. sincerity
D. refutation
E. canaraderie

Ans : B

43. INVETERATE
A. uninvited
B. illiterate
C. cumulative
D. beginning
E. incompetent

Ans : A

44. SATURNINE :
A. quick – wilted
B. genial
C. heavy – handed
D. distinguished
E. devout
45
Ans : E

45. PERSPICACIOUS :
A. Insufficiently precise
B. of indefinite duration
C. dull wilted
D. lacking intrinsic value
E. condemnatory

Ans : D

46. INCARCERATE :
A. summon
B. assist
C. liberate
D. anticipate
E. confide

Ans : C

47. INSOLVENCY :
A. ability to sustain growth
B. concentration
C. coherence
D. ability to pay one’s debts
E. compatibility

Ans : D

48. EFFLUVIA :
A. controlled reactions
B. predictable results
C. important examples
D. descried products
E. relevant theories

Ans : C

49. APPOSITE :
A. parallel
B. synonymous
C. hostile
D. inappropriate
E. vague

Ans : D

50. GRATUITOUS :
A. frank
B. pithy
C. warranted
D. frugal
E. ingenuous

Ans : A

51. PREFATORY :
A. intelligent
B. outstanding
46
C. predatory
D. conclusive
E. magnificent

Ans : E

52. CONCILIATE :
A. arrive
B. appeal
C. retaliate
D. estrange
E. lie

Ans : B

53. SUBSERVIENT :
A. fawning
B. obsequious
C. miserly
D. omnipresent
E. haughty

Ans : C

54. VAUNTED :
A. berated
B. belittled
C. lauded
D. wicked
E. worried

Ans : C

55. QUOTA :
A. Anonymous remark
B. decisive action
C. debatable issue
D. unlimited number
E. irrelevant topic

Ans : D

56. CONTENTIOUS :
A. satisfied
B. pacific
C. hungry
D. bellicose
E. dissatisfied

Ans : D

57. OBLOQUY :
A. fame
B. name
C. inquiry
D. shame
E. collogue

Ans : D
47
58. PENCHANT :
A. distaste
B. scabbard
C. agreement
D. earring
E. beginning

Ans : C

59. BALEFUL :
A. empty
B. tasty
C. gaudy
D. full
E. congenial

Ans : D

60. CURT :
A. contractual
B. precise
C. honest
D. voluble
E. peremptory

Ans : D

61. INVIDIOUS :
A. candid
B. stubborn
C. defensive
D. hostile
E. inoffensive

Ans : E

62. MACERATE :
A. cover by painting
B. assess by observing
C. harden by drying
D. influence by lying
E. cure by medicating

Ans : B

63. SKEPTICISM :
A. reason
B. conviction
C. plausibility
D. audricty
E. argument

Ans : D

64. IGNOMINIOUS :
A. scholarly
B. incognito
C. laudatory
D. disgraceful
E. erudite
48
Ans : B

65. CODA :
A. creflain
B. crescendo
C. prelude
D. improvisation
E. solo

Ans : A

66. PALTRY :
A. farm
B. scanty
C. excessive
D. friendly
E. benevolent

Ans : A

67. PUISSANCE :
A. strength
B. knowledge
C. liberality
D. skepticism
E. powerlessness

Ans : E

68. MANUMIT :
A. print
B. impress
C. enslave
D. fail
E. endeavor

Ans : D

69. GENUFLECT :
A. pronounce correctly
B. falsify
C. trick
D. stand erect
E. project

Ans : E

70. INNOCUOUS :
A. toxic
B. large
C. sober
D. impeccable
E. spotless

Ans : C

71. BAROQUE :
A. rococo
B. simple
C. common
49
D. stupid
E. boat like

Ans : B

72. MYOPIC :
A. blind
B. moral
C. visionary
D. farsighted
E. glassy

Ans : C

73. NASCENT :
A. loyal
B. fading
C. unnamed
D. treacherous
E. reoccuring

Ans : C

74. LOLL :
A. describe exactly
B. insist strongly
C. comply readily
D. notice incidentally
E. move vigorously

Ans : E

75. TURBULENCE :
A. immunity
B. tranquility
C. meditation
D. moderation
E. co – ordination

Ans : E

76. BANAL :
A. inclined
B. faithful
C. elaborate
D. forced
E. arresting

Ans : C

77. GERMINAL
A. sterilized
B. strategic
C. fully developed
D. primitive
E. excused .

Ans : D
50
78. GASCONADE
A. modesty
B. transparency
C. seizure
D. cleanliness
E. imposture

Ans : A

79. MIASMA
A. scenario
B. summing up
C. noxious fumes
D. fragrant aroma
E. benevolent

Ans : B

80. OPPORTUNIST
A. Man of principle
B. fatalist
C. fledgling
D. colleague
E. foe.

Ans : D

81. CENSURE
A. uncertainity
B. encomium
C. criticism
D. legal contual
E. matrimony

Ans : B

82. COMMODIOUS
A. product
B. space
C. cramped
D. company
E. roomy.

Ans : E

83. EFFRONTERY
A. modesty
B. confrontation
C. avoidance
D. shamelessness
E. impudence

Ans : B

84. OBSTREPEROUS
A. noisy
B. defiant
C. permeable
D. quiet
E. stubborn
51
Ans : E

85. PACIFY
A. ameliorate
B. patchup
C. truce
D. tormented
E. agitated

Ans : C

86. AMBIGUOUS
A. confusing
B. lucid
C. desirous
D. obfuscate
E. pun

Ans : A

87. MILITANT
A. Dramatic
B. combative
C. religious
D. pacific
E. quaint.

Ans : D

88. MOTILITY :
A. static
B. tension
C. ascent
D. liquidity
E. vulnerability

Ans : A

89. SINUOUS :
A. wet
B. vacant
C. numerous
D. direct
E. round

Ans : D

90. PLUMB :
A. reversed
B. horizontal
C. light
D. lofty
E. thin

Ans : B

91. SEGMENT:
A. inflate
B. affix
52
C. keep still
D. make whole
E. cleanse

Ans : D

92. OSSIFY :
A. reassemble fragments
B. overlook problems
C. create consensus
D. placate critics
E. transcend conventions

Ans : E

93. RAVEL :
A. increase in value
B. omit
C. remain silent
D. measure
E. knit

Ans : E

94. CALUMINATE :
A. vindicate
B. supplant
C. rejuvenate
D. follow
E. familiarize

Ans : A

95. TURPITUDE :
A. pragmatism
B. probity
C. judiciousness
D. animation
E. determinedness

Ans : B

96. INVECTIVE :
A. willing compliance
B. normality
C. restoration
D. fertility
E. laudatory words

Ans : E

97. PILLORY :
A. lament
B. foster
C. exalt
D. enjoy
E. forgive

Ans : C
53
98. UNTOWARD :
A. experienced
B. inevitable
C. industrious
D. straight forward
E. favourable

Ans : E

Quantitative Section : Quantitative Comparision

Directions:

In this section you will be given two quantities, one in column A and one in column B. You are to determine a
relationship between the two quantities and mark.

A. If the quantity in column A is greater than the quantity in column B.


B. If the quantity in column B is greater than the quantity in column A.
C. If the quantities are equal.
D. If the comparison cannot be determined from the information that is given.

1. Quantity A: (-6)4
Quantity B: (-6)5
A. if the quantity A is greater;
B. if the quantity B is greater;
C. if the two quantities are equal;
D. if the relationship cannot be determined from the information given.

Ans : A

2. Quantity A: Time to travel 95 miles at 50 miles per hour


Quantity B: Time to travel 125 miles at 60 miles per hour
A. Quantity A is greater
B. Quantity A equals Quantity B
C. Quantity B is greater
D. Relationship Indeterminate

Ans : C

3. Quantity A: (9/13)2
Quantity B: (9/13)1/2
A. Quantity A equals Quantity B
B. Relationship Indeterminate
C. Quantity B is greater
D. Quantity A is greater

Ans : C

4. Quantity A: 4 / 100
Quantity B: 0.012 / 3
A. Quantity B is greater
B. Quantity A equals Quantity B
C. Quantity A is greater
D. Relationship Indeterminate

Ans : C

5. x = 2y + 3
y = -2
54
Quantity A: x
Quantity B: -1

A. if the quantity in Column A is greater;


B. if the quantity in Column B is greater;
C. if the two quantities are equal;
D. if the relationship cannot be determined from the information given

Ans : C

6. x + 2y > 8

Quantity A: 2x + 4y
Quantity B: 20

A. if the quantity in Column A is greater;


B. if the quantity in Column B is greater;
C. if the two quantities are equal;
D. if the relationship cannot be determined from the information given.

Ans : D

7. Quantity A: The number of months in 7 years


Quantity B: The number of days in 12 weeks
A. if the quantity in Column A is greater;
B. if the quantity in Column B is greater;
C. if the two quantities are equal;
D. if the relationship cannot be determined from the information given

Ans : C

8. Quantity A: 1-1/27
Quantity B: 8/9 + 1/81
A. if the quantity in is greater;
B. if the quantity in is greater;
C. if the two quantities are equal;
D. if the relationship cannot be determined from the information given.

Ans : A

9. r/>s/>0/>

Quantity A: rs/r
Quantity B: rs/s

A. if the quantity A is greater;


B. if the quantity B is greater;
C. if the two quantities are equal;
D. if the relationship cannot be determined from the information given.

Ans : B

10. Quantity A: 0.83


Quantity B: 0.81/3
A. Quantity B is greater
B. Relationship Indeterminate
C. Quantity A is greater
D. Quantity A equals Quantity B

Ans : A
55
11. t is a positive integer.
4/7 = t/s

Quantity A: s
Quantity B:7

A. if the quantity in Column A is greater;


B. if the quantity in Column B is greater;
C. if the two quantities are equal;
D. if the relationship cannot be determined from the information given

Ans : D

12. Quantity A: (0.82)2(0.82)3


Quantity B:(0.82)6
A. if the quantity in Column A is greater;
B. if the quantity in Column B is greater;
C. if the two quantities are equal;
D. if the relationship cannot be determined from the information given.

Ans : A

13. For all real numbers a, let a* = 1 - a.

Quantity A: ((-1)*)*
Quantity B: 2*

A. if the quantity in Column A is greater;


B. if the quantity in Column B is greater;
C. if the two quantities are equal;
D. if the relationship cannot be determined from the information given.

Ans : C

14. Quantity A: (x - 1)(x)(x + 1)


Quantity B:(x)(x)(x)
A. if the quantity in Column A is greater;
B. if the quantity in Column B is greater;
C. if the two quantities are equal;
D. if the relationship cannot be determined from the information given.

Ans : D

15. Quantity A: (3 x 4 x 17) / (121 x 100)


Quantity B: (4 x 5 x 19) / (1000 x 121)
A. Quantity A is greater
B. Quantity A equals Quantity B
C. Relationship Indeterminate
D. Quantity B is greater

16. Consider a triangle PQR.

Quantity A: length of PQ + length of QR


Quantity B: length of PR

A. Quantity A is greater
B. Quantity B is greater
56
C. Relationship Indeterminate
D. Quantity A equals Quantity B

Ans : A

17. Quantity A: (27 - 13) (296 + 534)


Quantity B: (27 + 13) (534 + 296)
A. Quantity B is greater
B. Quantity A equals Quantity B
C. Relationship Indeterminate
D. Quantity A is greater

Ans : D

18. Quantity A: A = 1.1


Quantity B: 12.11/2
A. Relationship Indeterminate
B. Quantity B is greater
C. Quantity A equals Quantity B
D. Quantity A is greater

Ans : B

19. 100 < y < 200 and 100 < z < 210

Quantity A: y
Quantity B: z

A. Quantity A is greater
B. Quantity A equals Quantity B
C. Quantity B is greater
D. Relationship Indeterminate

Ans : D

20. y2 + z2 = 34 and yz = 15

Quantity A: y2 + 2yz + z2
Quantity B: (y + z)2

A. Quantity B is greater
B. Relationship Indeterminate
C. Quantity A is greater
D. Quantity A equals Quantity B

Ans : D

21. Consider a rectangle. The length of its shorter side is 8, and the length of its diagonal is 16.

Quantity A: 30o
Quantity B: measure of angle formed by diagonal and shorter side

A. Relationship Indeterminate
B. Quantity A equals Quantity B
C. Quantity A is greater
D. Quantity B is greater

Ans : D
57
22. Quantity A: (y + 5) 2

Quantity B: (y - 5)2
A. Quantity B is greater
B. Relationship Indeterminate
C. Quantity A equals Quantity B
D. Quantity A is greater

Ans : B

23. Quantity A: (1/25)1/2 + (1/144)1/2


Quantity B: [(1/25) + (1/144)]1/2
A. Relationship Indeterminate
B. Quantity A is greater
C. Quantity B is greater
D. Quantity A equals Quantity B

Ans : A

24. y2 + z2 = 34 and yz = 15

Quantity A: y2 + 2yz + z2
Quantity B: (y + z)2

A. Quantity A is greater
B. Relationship Indeterminate
C. Quantity A equals Quantity B
D. Quantity B is greater

Ans : C

25. 100 < y < 200 and 100 < z < 210

Quantity A: y
Quantity B: z

A. Quantity A is greater
B. Quantity A equals Quantity B
C. Quantity B is greater
D. Relationship Indeterminate

Ans : D

26. Quantity A: (y + 5)2


Quantity B: (y - 5)2
A. Quantity A equals Quantity B
B. Quantity A is greater
C. Relationship Indeterminate
D. Quantity B is greater

Ans : C

27. Consider a rectangle. The length of its shorter side is 8, and the length of its diagonal is 16.

Quantity A: 30o
Quantity B: measure of angle formed by diagonal and shorter side

A. Quantity A is greater
B. Quantity A equals Quantity B
C. Quantity B is greater
D. Relationship Indeterminate
58
Ans : C

28. The sum of three consecutive even numbers is 18.

Quantity A: Their average


Quantity B: 6

A. Relationship Indeterminate
B. Quantity A is greater
C. Quantity A equals Quantity B
D. Quantity B is greater

Ans : C

29. x - y > 10

Quantity A: y - x
Quantity B: 12

A. Quantity B is greater
B. Quantity A is greater
C. Quantity A equals Quantity B
D. Relationship Indeterminate

Ans : A

30. x = 0, y > 0

Quantity A: xy
Quantity B: yx

A. Quantity A equals Quantity B


B. Quantity A is greater
C. Quantity B is greater
D. Relationship Indeterminate

Ans : C

31.

Diagram is illustrative and is not drawn to scale.

Quantity A: Measure of angle 3 - Measure of angle 2


Quantity B: Measure of angle 5 - Measure of angle 6

A. Relationship Indeterminate
59
B. Quantity A equals Quantity B
C. Quantity B is greater
D. Quantity A is greater

Ans : B

32. Quantity A: 29
Quantity B: 92
A. Quantity B is greater
B. Quantity A is greater
C. Relationship Indeterminate
D. Quantity A equals Quantity B

Ans : B

33. 0 < -x < 10


11 < -y < 20

Quantity A: x
Quantity B: y

A. Relationship Indeterminate
B. Quantity A equals Quantity B
C. Quantity B is greater
D. Quantity A is greater

Ans : D

34.

Diagram is illustrative and is not drawn to scale.


Given angles 1 and 2 are equal,
length of side AB = x, length of side BC = y, length of side AC = z.

Quantity A: x + y
Quantity B: y + z

A. Quantity A is greater
B. Quantity A equals Quantity B
C. Quantity B is greater
D. Relationship Indeterminate

Ans : B
60

35.

Diagram is illustrative and is not drawn to scale.


In triangle ABC, AB = AC and measure of angle 1 = 100o.

Quantity A: Measure of angle 2 + Measure of angle 3


Quantity B: 90o

A. Quantity B is greater
B. Relationship Indeterminate
C. Quantity A is greater
D. Quantity A equals Quantity B

Ans : A

36. x and y are both positive and x/y > 5

Quantity A: 0.2x
Quantity B: y

A. Quantity A is greater
B. Quantity B is greater
C. Relationship Indeterminate
D. Quantity A equals Quantity B

Ans : A

37.

Diagram is illustrative and is not drawn to scale.


Given AB = AC and angle BAC = 60o

Quantity A: Length of side AB


Quantity B: Length of side BC

A. Quantity A equals Quantity B


B. Quantity B is greater
C. Relationship Indeterminate
61
D. Quantity A is greater

Ans : A

38. y2 = 36

Quantity A: y
Quantity B: 6

A. Relationship Indeterminate
B. Quantity A is greater
C. Quantity B is greater
D. Quantity A equals Quantity B

Ans : A

39.

Diagram is illustrative and is not drawn to scale.

Quantity A: Measure of angle 1 + Measure of angle 2 + Measure of angle 4


Quantity B: 180o

A. Relationship Indeterminate
B. Quantity A is greater
C. Quantity B is greater
D. Quantity A equals Quantity B

Ans : D

40.

Diagram is illustrative and is not drawn to scale.


In triangle ABC, angle A = 60o and AB = AC.

Quantity A: Measure of angle 1 + Measure of angle 2


Quantity B: 120o

A. Relationship Indeterminate
B. Quantity A is greater
C. Quantity A equals Quantity B
62
D. Quantity B is greater

Ans : C

41.

Diagram is illustrative and is not drawn to scale.

Quantity A: Measure of angle 2 + Measure of angle 3


Quantity B: 180o

A. Quantity B is greater
B. Quantity A is greater
C. Quantity A equals Quantity B
D. Relationship Indeterminate

Ans : C

42.

Diagram is illustrative and is not drawn to scale.


AB is the diameter of the circle.

Quantity A: Measure of angle 1


Quantity B: Measure of angle 2

A. Relationship Indeterminate
B. Quantity A is greater
C. Quantity B is greater
D. Quantity A equals Quantity B

Ans : D

43.
63
Diagram is illustrative and is not drawn to scale.

Quantity A: Measure of angle 1 + Measure of angle 3


Quantity B: Measure of angle 2 + Measure of angle 4

A. Relationship Indeterminate
B. Quantity A is greater
C. Quantity A equals Quantity B
D. Quantity B is greater

Ans : A

44.

Diagram is illustrative and is not drawn to scale.


In triangle ABC, AB = AC and measure of angle 1 = 100o.

Quantity A: Measure of angle 2 + Measure of angle 3


Quantity B: 90o

A. Quantity B is greater
B. Quantity A equals Quantity B
C. Relationship Indeterminate
D. Quantity A is greater

Ans : A

45.

Diagram is illustrative and is not drawn to scale.


Given angles 1 and 2 are equal,
length of side AB = x, length of side BC = y, length of side AC = z.

Quantity A: x + y
Quantity B: y + z

A. Quantity B is greater
B. Quantity A equals Quantity B
64
C. Quantity A is greater
D. Relationship Indeterminate

Ans : B

46. x and y are both positive and x/y > 5

Quantity A: 0.2x
Quantity B: y

A. Quantity B is greater
B. Relationship Indeterminate
C. Quantity A equals Quantity B
D. Quantity A is greater

Ans : D

47. yz < 0

Quantity A: (y - z)2
Quantity B: y2 + z2

A. Quantity A is greater
B. Quantity B is greater
C. Quantity A equals Quantity B
D. Relationship Indeterminate

Ans : A

48. For any positive integer n,


n! is the product of all positive integers less than or equal to n.

Quantity A: 20! / 17!


Quantity B: 80! / 78!

A. Quantity A is greater
B. Quantity B is greater
C. Quantity A equals Quantity B
D. Relationship Indeterminate

Ans : A

49. 2 < z < 4

Quantity A: π2z3
Quantity B: π3z2

A. Quantity A is greater
B. Quantity B is greater
C. Quantity A equals Quantity B
D. Relationship Indeterminate

Ans : D

50. Amy, Beth and Charlie divided a pizza amongst themselves.


Amy took 30% of the pizza and ate (3/4) of what she took.
Beth took 20% of the pizza.
Charlie ate (2/5) of what he took.
65
Quantity A: The amount Amy ate
Quantity B: The amount Charlie ate

A. Quantity A is greater
B. Quantity B is greater
C. Quantity A equals Quantity B
D. Relationship Indeterminate

Ans : A

51. p > 0 > q

Quantity A: p + q
Quantity B: pq

A. The quantity in Column A is greater.


B. The quantity in Column B is greater.
C. The quantities are equal.
D. The relationship cannot be determined from the information given.

Ans : D

52. Quantity A: The average (arithmetic mean) of x and y


Quantity B: The average (arithmetic mean) of x - 1 and y + 1
A. The quantity in Column A is greater.
B. The quantity in Column B is greater.
C. The quantities are equal.
D. The relationship cannot be determined from the information given.

Ans : C

53. The integer (x - 1) is a prime number between 40 and 50.

Quantity A: The sum of all different prime factors of x


Quantity B: 14

A. The quantity in Column A is greater.


B. The quantity in Column B is greater.
C. The quantities are equal.
D. The relationship cannot be determined from the information given.

Ans : B

54. A < C
B>D>0

Quantity A: A - B
Quantity B: C - D

A. The quantity in Column A is greater.


B. The quantity in Column B is greater.
C. The quantities are equal.
D. The relationship cannot be determined from the information given.

Ans : B

55. In a particular jellybean jar, the number of red jellybeans exceeds the number of white ones by a ratio of 3:2.
If two red jellybeans were removed, the ratio of red to white jellybeans would be 1:1.
66
Quantity A: The number of white jellybeans in the jar
Quantity B: 4

A. The quantity in Column A is greater.


B. The quantity in Column B is greater.
C. The quantities are equal.
D. The relationship cannot be determined from the information given.

Ans : C

Quantitative Section : Quantitative Ability

Directions:In this section you will be given two quantities, one in column A and one in column B. You are to
determine a relationship between the two quantities and mark.

A. If the quantity in column A is greater than the quantity in column B.


B. If the quantity in column B is greater than the quantity in column A.
C. If the quantities are equal.
D. If the comparison cannot be determined from the information that is given.

1. A rectangle is 14 cm long and 10 cm wide. If the length is reduced by x cms and its width is increased also
by x cms so as to make it a square then its area changes by :
A. 4
B. 144
C. 12
D. 2
E. None of the above.

Ans : A

2. A motorcycle stunts man belonging to a fair, rides over the vertical walls of a circular well at an average
speed of 54 kph for 5 minutes. If the radius of the well is 5 meters then the distance traveled is:
A. 2.5 kms
B. 3.5 kms
C. 4.5 kms
D. 5.5 kms
E. None of the above

Ans : C

3. If 1 cm on a map corresponds to an actual distance of 40 kms. And the distance on the map between
Bombay and Calcutta is 37.5 cms., the actual distance between them is :
A. 375 kms
B. 3750 kms
C. 1500 kms
D. 1375 kms
E. None of the above

Ans : C

4. A box contains 90 mts each of 100 gms and 100 bolts each of 150 gms. If the entire box weighs 35.5 kg.,
then the weight of the empty box is :
A. 10 kg
B. 10.5 kg
C. 11 kg
D. 11.5 kg
E. None of the above

Ans : D

5. If the radius of a circle is increased by 20% then the area is increased by :


67
A. 44%
B. 120%
C. 144%
D. 40%
E. None of the above

Ans : A

6. Tom, Dick and Harry went for lunch to a restaurant. Tom had $100 with him, Dick had $60 and Harry had
$409. They got a bill for $104 and decided to give a tip of $16. They further decided to share the total
expenses in the ratio of the amounts of money each carried. The amount of money which Tom paid more
than what Harry paid is
A. 120
B. 200
C. 60
D. 24
E. 36

Ans : E

7. A plot of land is in the shape of a trapezium whose dimensions are given in the figure below :

Hence the perimeter of the field is


A. 50 m
B. 64 m
C. 72 m
D. 84 m
E. None of the above

Ans : c

8. Four concentric ( having the same center ) circles with radii, x, 2x, 3x and 4x are drawn to form two rings A
and B as shown in the figure.

Ratio of the area of inner ring A to the area of outer ring B is


A. 1 : 2
B. 1 : 4
C. 2 : 3
D. 3 : 7
E. None of the above

Ans : D

9. If 3/p = 6 and 3/q = 15 then p - q = ?


A. 1/3
B. 2/5
68
C. 3/10
D. 5/6
E. None of the above

Ans : C

10. A father is three times as old as his son. After fifteen years the father will be twice as old as his son's age at
that time. Hence the father's present age is
A. 36
B. 42
C. 45
D. 48
E. None of the above

Ans : C

11. (1/4)3 + (3/4)3 + 3(1/4)(3/4)(1/4 + 3/4) =?


A. 1/64
B. 27/64
C. 49/64
D. 0
E. 1

Ans : E

12. If the area of two circles are in the ratio 169 : 196 then the ratio of their radii is
A. 10 : 11
B. 11 : 12
C. 12 : 13
D. 13 : 14
E. None of the above

Ans : D

13. A semi-circle is surmounted on the side of a square. The ratio of the area of the semi-circle to the area of the
square is

A. 1:2
B. 2:p
C. p:8
D. 8:p
E. None of the above

Ans : C

14. Which of the following is the greatest ?


A. 40% of 30
B. 3/5 of 25
C. 6.5% of 200
D. Five more than the square of 3
E. 1/2-4
69
Ans : E

15. Two identical taps fill 2/5 of a tank in 20 minutes. When one of the taps goes dry in how many minutes will
the remaining one tap fill the rest of the tank ?
A. 5 minutes
B. 10 minutes
C. 15 minutes
D. 20 minutes
E. None of the above

Ans : C

16. If the value of XYZ Company stock drops from $25 per share to $21 per share, what is the percent of the
decrease?
A. 4
B. 8
C. 12
D. 16
E. 20

Ans : D

17. If a building b feet high casts a shadow f feet long, then, at the same time of day, a tree t feet high will cast a
shadow how many feet long?
A. ft/b
B. fb/t
C. b/ft
D. tb/f
E. t/fb

Ans : A

18. If x, y, and z are consecutive negative integers, and if x > y > z, which of the following must be a positive odd
integer?
A. xyz
B. (x - y) (y - z)
C. x - yz
D. x(y + z)
E. x + y + z

Ans : B

19. At a certain ice cream parlor, customers can choose among five different ice cream flavors and can choose
either a sugar cone or a waffle cone. Considering both ice cream flavor and cone type, how many distinct
triple-scoop cones with three different ice cream flavors are available?
A. 12
B. 16
C. 20
D. 24
E. 30

Ans : C

20. What is the greatest value of a positive integer n such that 3n is a factor of 1815?
A. 15
B. 18
C. 30
D. 33
E. 45

Ans : C
70
21. If .2t = 2.2 - .6s and .5s = .2t + 1.1, then s =
A. 1
B. 3
C. 10
D. 11
E. 30

Ans : B

22. Five years ago, Beth's age was three times that of Amy. Ten years ago, Beth's age was one half that of
Chelsea. If C repre- sents Chelsea's current age, which of the following represents Amy's current age?
A. c/6 + 5
B. 2c
C. (c-10)/3
D. 3c-5
E. 5c/3 - 10

Ans : A

23. A portion of $7200 is invested at a 4% annual return, while the remainder is invested at a 5% annual return.
If the annual income from both portions is the same, what is the total income from the two investments?
A. $160
B. $320
C. $400
D. $720
E. $1,600

Ans : B

24. An empty swimming pool can be filled to capacity through an inlet pipe in 3 hours, and it can be completely
drained by a drainpipe in 6 hours. If both pipes are fully open at the same time, in how many hours will the
empty pool be filled to capacity?
A. 4
B. 4.5
C. 5
D. 5.5
E. 6

Ans : E

25. If r = (3p + q)/2 and s = p - q, for which of the following values of p would r2 = s2?
A. 1q/5
B. 10 - 3q/2
C. q - 1
D. 3q
E. 9q/2 - 9

Ans : A

26. At 10 a.m. two trains started traveling toward each other from stations 287 miles apart. They passed each
other at 1:30 p.m. the same day. If the average speed of the faster train exceeded the average speed of the
slower train by 6 miles per hour, which of the following represents the speed of the faster train, in miles per
hour?
A. 38
B. 40
C. 44
D. 48
E. 50

Ans : C
71
27. On the xy-coordinate plane, points A and B both lie on the circumference of a circle whose center is O, and
the length of AB equals the circle's diameter. If the (x,y) coordinates of O are (2,1) and the (x,y) coordinates
of B are (4,6), what are the (x,y) coordinates of A?
A. (3, 3/2)
B. (1, 2/2)
C. (0, -4)
D. (2/2, 1)
E. (-1, -2/2)

Ans : C

28. If a rectangle's length and width are both doubled, by what percent is the rectangle's area increased?
A. 50
B. 100
C. 200
D. 300
E. 400

Ans : D

29. A rectangular tank 10" by 8" by 4" is filled with water. If all of the water is to be transferred to cube-shaped
tanks, each one 3 inches on a side, how many of these smaller tanks are needed?
A. 9
B. 12
C. 16
D. 21
E. 39

Ans : B

30. Point Q lies at the center of the square base (ABCD) of the pyramid pictured above. The pyramid's height
(PQ) measures exactly one half the length of each edge of its base, and point E lies exactly halfway
between C and D along one edge of the base. What is the ratio of the surface area of any of the pyramid's
four triangular faces to the surface area of the shaded triangle?
A. 3 :√2
B. √5:1
C. 4√3:3
D. 2√2:1
E. 8:√5

Ans : D

Questions 1 - 5 refers to the following table:

PROFILE OF CONGRESS IN YEAR X


(total membership: 535)

House of
Senate
Representatives
Party
292 Democratic 62
143 Republican 38
435 TOTAL 100
Sex
418 Male 100
17 Female 0
Age
27 Youngest 34
77 Oldest 80
Average
48 54
(arithmetic mean)
Religion
72
255 Protestant 69
107 Catholic 12
18 Jewish 5
4 Mormon 3
51 Other 11
House of
Senate
Representatives
Profession
215 Lawyer 63
Business Executive
81 15
or Banker
45 Educator 6
14 Farmer or Rancher 6
22 Career Government 0
24 Official 4
Journalist or
2 0
Communications
1 Executive 1
0 Physician 2
Veterinarian
6 0
Geologist
Worker or Skilled
25 Tradesperson 3
Other
Ethnic Group
17 Black American 1
2 Asian American 3
4 Hispanic American 0

1. In the Senate, if 25 male members were replaced by 25 female members, the ratio of male members to
female members would be
A. 4 to 1
B. 3 to 1
C. 3 to 2
D. 2 to 1
E. 1 to 1

Ans : B

2. Approximately what percent of the members of Congress are lawyers?


A. 63%
B. 58%
C. 56%
D. 52%
E. 49%

Ans : D

3. If 5 senators are Catholic Democrats, how many senators are neither Catholic nor Democratic?
A. 79
B. 74
C. 69
D. 31
E. 21

Ans : D

4. If all lawyers and all women in the House of Representatives vote for the passage of a bill, how many more
votes will be needed for a majority?
A. 435
B. 220
C. 3
D. 0
E. It cannot be determined from the information given.
73
Ans : E

5. Which of the following can be inferred from the information given in the chart?

I. More than 80 percent of the men in Congress are members of the House of Representatives.
II. The percent of members who are categorized as farmers or ranchers is greater for the House of
Representatives than for the Senate.
III. The median age in the Senate is 57.

A. I only
B. II only
C. III only
D. I and II
E. I and III

Ans : A

Questions 6 - 7 refers to the following table:

PERCENT CHANGE IN DOLLAR AMOUNT OF SALES


IN CERTAIN RETAIL STORES FROM 1977 TO 1979

Percent Change

From 1977 From 1978


Store
to 1978 to 1979
P +10 -10
Q -20 +9
R +5 +12
S -7 -15
T +17 -8

1. In 1979, for which of the stores was the dollar amount of sales greater than that of any of the others shown?
A. P
B. Q
C. R
D. S
E. It cannot be determined from the information given.

Ans : E

2. In store T, the dollar amount of sales for 1978 was approximately what percent of the dollar amount of sales
for 1979?
A. 86%
B. 92%
C. 109%
D. 117%
E. 122%

Ans : C

Questions 8 - 9 refers to the following Figure:


74

8. Of every dollar received by the federal government, how much (in cents) is from coporate sources?
A. 32
B. 70
C. 30
D. 35
E. 29

Answer : 1

9. what percentage of the federal revenue is derived from borrowings?


A. 0.2%
B. 0.02%
C. 2.7%
D. 1.2%
E. 2.5%

Answer : 3

Questions 10 - 11 refers to the following table:

DIRECTIONS: The following question are based on the bellow table, which shows per capita Mean Expenditure, Per
capita Food expenditure, Number of Households and Per capita cereal consumption, in both quantity and value, for
different expenditure classes of rural India. The sampled 41597 households are divided into 12 expenditure classes,
starting from less than Rs.65 per month per capita and ending at more than Rs.385 per capita per month.

10. According to the results of this sample survey, what is the proportion of total expenditure on food to total
expenditure for all the sampled households taken together?
A. 58%
75
B. 36.7%
C. 63.3%
D. 71%
E. Cannot be determined

Answer : 3

11. What is the difference, approximately, between the gross expenditure of the sampled households in the
Rs.95-110 expenditure class and in the Rs.180-215 expenditure class?
A. 372000
B. 448000
C. 496000
D. 93.8
E. 52.3

Answer : A

Questions 12 - 13 refers to the following Graph:

GRAPH SHOWS EXPENDITURE ON ARMS BY DIFFERENT COUNTRIES (VALUE IN DOLLARS '000 MILLIONS)

12. The amount spent by country C in 1983 is what percentage more than the amount spent by Countries A and
B together in 1977? (Find approximately)
A. 50%
B. 179%
C. 75%
D. 13%
E. 70%

Answer : C

13. Which of the following statements must be true?

i. Country A spends minimum amount of its budget on arms.


ii. Throughout, Country C has spent the maximum amount on arms during the years shown.
iii. An examination of the information for the last 3 years reveals that generally all 3 countries are
reducing their expenditure on arms.
76
A. i only.
B. i and ii only
C. i and iii only
D. ii and iii only
E. None of the statements above.

Answer : E

Analytical Section : Analytical Reasoning

Directions :All GRE sample analytical reasoning questions are based on a passage or set of conditions. While
answering a few of the GRE sample analytical reasoning questions, you would find it useful to draw a rough diagram.
To answer any question choose the answer you think is most appropriate among the given options.

Questions 1- 3

Three men (Tom, Peter and Jack) and three women (Eliza, Anne and Karen) are spending a few months at a hillside.
They are to stay in a row of nine cottages, each one living in his or her own cottage. There are no others staying in
the same row of houses.

1. Anne, Tom and Jack do not want to stay in any cottage, which is at the end of the row.
2. Eliza and Anne are unwilling to stay besides any occupied cottage..
3. Karen is next to Peter and Jack.
4. Between Anne and Jack’s cottage there is just one vacant house.
5. None of the girls occupy adjacent cottages.
6. The house occupied by Tom is next to an end cottage.

1. Which of the above statements can be said to have been derived from two other statements ?
A. Statement 1
B. Statement 2
C. Statement 3
D. Statement 5
E. Statement 6

Ans : D

2. How many of them occupy cottages next to a vacant cottage ?


A. 2
B. 3
C. 4
D. 5
E. 6

Ans : C

3. Which among these statement(s) are true ?

I. Anne is between Eliza and Jack.


II. At the most four persons can have occupied cottages on either side of them. .
III. Tom stays besides Peter.

A. I only
B. II only
C. I and III only
D. II and III only
E. I, II and III

Ans : C

Questions 4 - 7
77
An employee has been assigned the task of allotting offices to six of the staff members. The offices are numbered 1 -
6. The offices are arranged in a row and they are separated from each other by six foot high dividers. Hence voices,
sounds and cigarette smoke flow easily from one office to another.

Miss Robert's needs to use the telephone quite often throughout the day. Mr. Mike and Mr. Brown need adjacent
offices as they need to consult each other often while working. Miss. Hardy, is a senior employee and has to be
allotted the office number 5, having the biggest window. .

Mr. Donald requires silence in the offices next to his. Mr. Tim, Mr. Mike and Mr. Donald are all smokers. Miss Hardy
finds tobacco smoke allergic and consecutively the offices next to hers to be occupied by non-smokers.

Unless specifically stated all the employees maintain an atmosphere of silence during office hours.

4. The ideal candidate to occupy the office furthest from Mr. Brown would be
A. Miss Hardy
B. Mr. Mike
C. Mr. Tim
D. Mr. Donald
E. Mr. Robert

Ans : D

5. The three employees who are smokers should be seated in the offices.
A. 1, 2 and 4
B. 2, 3 and 6
C. 1, 2 and 3
D. 1, 2 and 3
E. 1, 2 and 6

Ans : D

6. The ideal office for Mr. Mike would be.


A. 2
B. 6
C. 1
D. 3
E. 4

Ans : D

7. In the event of what occurrence, within a period of one month since the assignment of the offices, would a
request for a change in office be put forth by one or more employees ?
A. Mr. Donald quitting smoking.
B. The installation of a noisy teletype machine by Miss Hardy in her office.
C. Mr. Robert’s needing silence in the office (s) next to her own. .
D. Mr. Brown suffering from laryngitis.
E. Mr. Tim taking over the duties formerly taken care of by Miss. Robert. .

Ans : E

Questions 8 - 10

In an experiment conducted at a laboratory, 160 white mice were injected with Serum D. 160 other white mice were
injected with a harmless sugar solution .In two weeks time 39% of the white mice, who were injected with Serum D
contracted the highly contagious and often fatal disease, jungle fever. Hence, it can be concluded that jungle fever is
caused by some elements similar to the elements in Serum D.

8. The above discussion would be weakened most severely in case it is shown that
A. People contracting jungle fever are usually the victims of the bite of the South American Lesser
Hooded Viper.
78
B. One among the 160 white mice had already contracted jungle fever prior to the laboratory
experiment.
C. The natural habitats of white mice does not contain any of the elements found in Serum D.
D. The scientists administered the injections being ignorant of the contents of the solutions used.
E. The 160 white mice used in the laboratory experiment were kept isolated from each other.

Ans : B

9. The above argument would be highly empowered in case it were shown that:
A. Some of the elements in Serum D are extracted from the root of a certain poisonous jungle
wildflower.
B. Within a period of two weeks about 40% of the white mice, who were injected with a harmless
sugar solution also contracted jungle fever.
C. Almost all the white mice died within a period of two days after the first symptoms appeared.
D. Normally the rate of jungle fever among white mice is less than 0.01%.
E. Invariably the blood of the victims of jungle fever victims contains a high level of a certain toxic
substance also found in serum D.

Ans : E

10. Distribution of leaflets and delivering speeches on government property should be outlawed. Radicals and
fanatics have no right to use public property when peddling their unsavory views.

The argument above is based on the postulate

A. The general public has a special concern in the free exchange of different political views.
B. Radicals and fanatics prefer the use of public property while propagating their viewpoint.
C. Every person who hands out leaflets and delivers speeches is a radical or fanatic.
D. Legal constraints which are applicable to one group need not be equally applicable to all.
E. Any political activity, which hinders the proper functioning of the government should not be
protected by the law.

Ans : C

Questions 11 - 12

Successfully adjusting to one's environment leads to happiness. War at a universal level war destroys the weaker
people, who are the most unable to adjust to their environment. Thus, war at the universal level puts weaklings out of
their misery and allows more space for their predators to enjoy life in a better manner. As those actions have to be
performed, which maximize the level of happiness of the greatest number, war at a universal level should take place.

11. What response would the author of the above discussion come up with, in the case of the objection that the
weaklings far exceed strong people?
I. He would respond with the statement that the person making the objection is a weakling.
II. He would respond by saying that weaklings will be miserable no matter what happens.
III. He would respond with the statement that the strong would be frustrated if the weaklings are
destroyed.

A. I only
B. II only
C. III only
D. I and II only
E. II and III only

Ans : E

12. The author's discussion would be greatly if he agreed to which of the following?

I. Technology could change the environment.


II. War at the universal level would be an integral part of the environment.
III. It is possible for the strong to survive without suppressing the weak.
79
A. I only
B. II only
C. III only
D. I and III only
E. I, II and III only

Ans : A

Questions 13 - 15

Come back with us to the real America leaving behind the turmoil of civilization. The real America is still inhabited by
the eagle, the buffalo, the mountain lion and elk; it is still spacious, sprawling and majestic. Experience the freedom
and serenity still to be found in

13. Choose the best option to complete the above statement:


A. the natural beauty of our land
B. the fascinating urban centers
C. the wild terrain of Africa
D. one's own subconscious
E. the great sprawling cities of the Southwest

Ans : A

14. The above paragraph is most likely to appear in which of the following?
A. A Hunter's Guide to The United States
B. Exploring the Great Outdoors
C. The Quiet Beauty of Alaska
D. How the Eagle Became Extinct
E. Returning to America

Ans : D

15. When I am elected, I will work towards effecting those changes for which I have been fighting all these
years. We will work together to do away with the bureaucratic bogs which have existed ever since my
opponent took office. Everyone of you knows what I stand for; I invite my opponent to ...

For completion of the above statement choose the best option:

A. hand in his resignation graciously


B. make his stance clear
C. stop lying to the public
D. get our city more federal aid
E. extend his support to me

Ans : B

16. We can never make our beliefs regarding the world certain. Even scientific theory of a most rigorous and
well-confirmed nature is likely to change over a decade or even tomorrow. If we refuse to even try to
understand, then it is like resigning from the human race. Undoubtedly life of an unexamined kind is worth
living in other respects--as it is no mean thing to be a vegetable or an animal. It is also true that a man
wishes to see this speculative domain beyond his next dinner.

From the above passage it is clear that the author believes that

A. men would not do well to speculate


B. progress in the scientific field is impossible
C. one should live life with the dictum 'what will be will be'
D. men should ignore their animal needs
E. men are different from animals as far as their reasoning abilities are concerned.
80
Ans : E

Questions 17-21

Two or more essences out of a stock of five essences-- L, M, N, O, and P are used in making all perfumes by a
manufacturer. He has learned that for a blend of essences to be agreeable it should comply with all the rules listed
below.

• A perfume containing L, should also contain the essence N, and the quantity of N should be twice as that of
L.
• A perfume containing M, must also have O as one of its components and they should be in equal proportion.
• A single perfume should never contain N as well as O.
• O and P should not be used together.
• A perfume containing the essence P should contain P in such a proportion that the total amount of P present
should be greater than the total amount of the other essence or essences used.

17. Among the following which is an agreeable formula for a perfume?


A. One part L, one part P
B. Two parts M, two parts L
C. Three parts N, three parts L
D. Four parts O, four parts M
E. Five parts P, five parts M

Ans : D

18. Adding more amount of essence N will make which of the following perfumes agreeable?
A. One part L, one part N, five parts P
B. Two parts M, two parts N, two parts P
C. One part M, one part N, one part P
D. Two parts M, one part N, four parts P
E. Two parts N, one part O, three parts P

Ans : A

19. Among the following, the addition of which combination would make an unagreeable perfume containing two
parts N and one part P agreeable?
A. One part L
B. One part M
C. Two parts N
D. One part O
E. Two parts P

Ans : E

20. Among the following which combination cannot be used together in an agreeable perfume containing two or
more essences?
A. L and M
B. L and N
C. L and P
D. M and O
E. P and N

Ans : A

21. Among the below mentioned formulas, which can be made agreeable by the eliminating some or all of one
essence ?
A. One part L, one part M, one part N, four parts P
B. One part L, two parts N, one part O, four parts P
C. One part L, one part M, one part O, one part P
D. Two parts L, two parts N, one part O, two parts P
E. Two parts M, one part N, two parts O, three parts P
81
Ans : B

22. Everything that a person does, which is dictated by reason of ignorance is not voluntary. Involuntary actions
are those which produce pain and repentance. Incase a man has done something in his ignorance and he
does not feel vexed due to his action, he has not acted voluntarily as he was not aware of what he was
doing, nor yet involuntarily since he is not pained.

After reading this passage we can arrive at the conclusion that:

A. A person is not a voluntary agent, if he acts by reason of ignorance and repents. .


B. If an action is done by reason of ignorance and is not voluntary , then it was repented.
C. A man is an involuntary agent, if he acts by reason of ignorance.
D. Some actions are either involuntary or not voluntary.
E. If a man is not a voluntary agent, then he acted by reason of ignorance and repents.

Ans : A

23. Everything that God knows necessarily is, because even what we ourselves know necessarily is; and, of
course, our knowledge is not as certain as God's knowledge is. But no future contingent thing necessarily
is..

Among the following statements, which naturally follows from the above:

1. There are no future contingent things.


2. It is not true that God has knowledge of only necessary things.
3. God has knowledge of no contingent future things.
4. It is not possible for us to know God.
5. God has knowledge of everything. .

Ans : C

Questions 24 - 25

Some lawyers are of the view that the observation of the intrinsic qualities of pornography in any composition
depends on literary criticism and hence it is a matter of opinion. It is rather odd, though, that in a legal connection,
serious critics themselves quite often behave as if they believed criticism to be a matter of opinion. Why be a critic -
and teach in universities - in case criticism involves nothing but uttering capricious and arbitrary opinions ?

24. In the above argument the author is trying to establish that


A. whether a composition can be called pornographic or not is a matter of opinion. .
B. it is not a matter of opinion whether a work is pornographic.
C. observance of the qualities of pornography is not dependent on literary criticism.
D. critics seem hypocritical.
E. critics should not teach at universities.

Ans : D

25. The above discussion would be weakened if it is pointed out that:


A. literary critics are of the opinion that nothing is pornographic.
B. lawyers believe that the observance of the qualities of pornography is a matter of opinion, as
literary critics are not in agreement in this regard.
C. literary critics are not legal authorities.
D. literary critics should not concern themselves with deciding what is pornographic.
E. literary critics in the teaching profession at the university level are init only for the money.

Ans : B

Questions 26 - 31

Nine individuals - Z, Y, X, W, V, U, T, S and R - are the only candidates, who can serve on three committees-- A, B
and C, and each candidate should serve on exactly one of the committees.
82
Committee A should consist of exactly one member more than committee B.
It is possible that there are no members of committee C.
Among Z, Y and X none can serve on committee A.
Among W, V and U none can serve on committee G.
Among T, S and R none can serve on committee C.

26. In case T and Z are the individuals serving on committee B, how many of the nine individuals should serve
on committee C?
A. 3
B. 4
C. 5
D. 6
E. 7

Ans : B

27. Of the nine individuals, the largest number that can serve together on committee C is
A. 9
B. 8
C. 7
D. 6
E. 5

Ans : D

28. In case R is the only individual serving on committee B, which among the following should serve on
committee A?
A. W and S
B. V and U
C. V and T
D. U and S
E. T and S

Ans : E

29. In case any of the nine individuals serves on committee C, which among the following should be the
candidate to serve on committee A?
A. Z
B. Y
C. W
D. T
E. S

Ans : C

30. In case T, S and X are the only individuals serving on committee B, the total membership of committee C
should be:
A. Z and Y
B. Z and W
C. Y and V
D. Y and U
E. X and V

Ans : A

31. Among the following combinations which could constitute the membership of committee C?
A. Y and T
B. X and U
C. Y, X and W
D. W, V and U
E. Z, X, U and R
83
Ans : B

Questions 32 - 34

(M, N, O and P are all different individuals)

I. M is the daughter of N.
II. N is the son of O
III. O is the father of P.

32. Among the following statements, which is true ?


A. O is the uncle of M.
B. P and N are brothers
C. M is the daughter of P.
D. If B is the daughter of N, then M and B are sisters.
E. If C is the granddaughter of O, then C and M are sisters.

Ans : D

33. Which among the following statements is contradictory to the above premises?
A. P is the father of M.
B. O has three children.
C. M has one brother.
D. M is the granddaughter of O.
E. Another party C, could be the mother of M.

Ans : A

34. If B is the son of N and B has one brother, D, then

I. M is the sister of D.
II. D and N are brothers.
III. O is the grandfather of D.

A. I only
B. II only
C. III only
D. I and III only
E. I and II only

Ans : D

Questions 35 - 41

The only people to attend a conference were four ship captains and the first mates of three of those captains. The
captains were L, M, N and O; the first mates were A, D and G. Each person in turn delivered a report to the assembly
as follows:

Each of the first mates delivered their report exactly after his or her captain. The first captain to speak was M, and
captain N spoke after him.

35. Among the following which is not an appropriate order of delivered reports?
A. M, A, N, G, O, L, D
B. M, D, N, G, L, O, A
C. M, N, A, L, D, O, G
D. M, N, A, O, D, L, G
E. M, N, G, D, O, L, A

Ans : E
84
36. In case L speaks after A, and A is the third of the first mates to speak, then among the following statements
which would be untrue?
A. O spoke immediately after G.
B. The order of the first four speakers was M, G, N, D.
C. O's first mate was present.
D. A was the fourth speaker after M.
E. The captains spoke in the order M, N, O, L.

Ans : D

37. Among the following statements which statement must be true?


A. In case the second speaker was a captain, the seventh speaker was a first mate.
B. In case the second speaker was a first mate, the seventh speaker was a captain.
C. In case the third speaker was a first mate, the seventh speaker was a captain.
D. In case the third speaker was a captain, the seventh speaker was a first mate.
E. In case the seventh speaker was a first mate, the first and third speakers were captains.

Ans : A

38. In case A spoke immediately after L and immediately before O, and O was not the last speaker, L spoke
A. second
B. third
C. fourth
D. fifth
E. sixth

Ans : C

39. In case G is M's first mate, D could be the person who spoke immediately
A. prior to T
B. prior to L
C. prior to V
D. after T
E. after V

Ans : D

40. In case A is the third of the first mates to speak, and L is the captain whose first mate is not present, which
among the following statements must be true?
A. A spoke sometime before L.
B. D spoke sometime before O.
C. L spoke sometime before O.
D. O spoke sometime before L.
E. O spoke sometime before N.

Ans : B

41. Among the following statements, which would make M, D, N, G, L, O, A the only possible sequence of
speakers?
A. D is M's first mate; G is N's first mate; A is O's first mate.
B. D is M's first mate; G is N's first mate; A was the second to speak after L.
C. The order of the first four speakers was M, D, N, G.
D. The order of the last three speakers was L, O, A.
E. The order in which the captains spoke was M, N, L, O.

Ans : B

Questions 42 - 45

1. The microbe A causes the contagious disease A.


2. The first symptoms appear after a period of two days since the microbe A enters the body.
85
3. The microbe A is found in some flies and bees.
4. A fly bit Jack on Monday, February 6.
5. Nick worked with Jack the next day, Tuesday, February 7.

There were no other possibilities of exposure to Disease A.

42. In case Jack showed symptoms of Disease A, which of the following statements would be true?
I. Jack contracted the Disease A from Nick.
II. Jack first noticed symptoms of Disease A on February 8.
III. The fly that Jack was bitten by was not a carrier of the microbe A.

A. I only
B. II only
C. III only
D. I and II only
E. I and III only

Ans : B

43. In case Nick displayed symptoms of the disease A, which among the following would be true?
A. I only
B. II only
C. III only
D. II and III only
E. I, II and III

Ans : D

44. In case Jack displayed symptoms of Disease A, which would be true?

I. Jack was also bitten by a fly on February 5.


II. Jack was bitten by a mosquito which carried the microbe A.
III. Nick contracted Disease A from Jack.

1. I only
2. II only
3. III only
4. I and II only
5. II and III only

Ans : D

43. In case Nick displayed the symptoms of Disease A which would be true?

I. Nick was bitten by a bee on February 6.


II. Nick ate food which contained the microbe A.
III. Nick also worked with Jack on February 6.

1. I only
2. II only
3. III only
4. I and II only
5. I, II and III

Ans : A

Questions 46 - 47

The principal evil in today's society is selfishness. Everywhere we see people, who are concerned only with
themselves. Personal advancement is the only motivating force in the world today. This does not mean that
86
individuals are not willing to help one another; on the contrary, _________. But, these are only short-term
occurrences which ultimately serve our long-term goal of personal gain.

46. To fill in the blank in the above passage, select one of the options from the below mentioned options:
1. we are always trying to undermine others’ endeavors.
2. my uncle Jeremy used to help me with my homework.
3. no one can be trusted, not even close friends.
4. our yearning for power prevents us from understanding our existential purpose.
5. there are many occasions when we graciously offer our assistance.

Ans : E

47. Which among the following options would most strongly contradict the author's attitude towards society?
1. The greatest strength of society is altruism.
2. The forces of good will ultimately triumph over evil.
3. Our short-term actions may ostensibly contradict our long-term goals.
4. We must all learn the art of selfishness.
5. Morality is the bedrock of a growing community.

Ans : A

48. It can be proved by an example that our words are devoid of meaning as they cannot be distinguished from
their opposites. People think that they are aware of the difference between the meanings of 'bald' and
'having hair' Let us suppose that an average twenty-one year old has X strands of hair on his or her head.
We say that such a person is not bald but has hair. But surely one hair less would make no difference, and a
person with X - 1 hairs on his or her head would be said to have hair. Suppose we go on like this,
decreasing one hair at a time, the result would be the same. Then what difference would there be between
someone who has one hair and someone who has none? We call them both bald. Thus, we cannot make a
distinction between the terms 'bald' and 'having hair.'

Among the following statements, which statement best counters the argument above?

A. The word 'bald' can be translated into other languages.


B. A word can have more than one meaning.
C. A word such as 'monkey' can be applied to several animals that differ in some respects.
D. Words can lack precision without being meaningless.
E. People cannot think clearly without using words.

Ans : D

49. Virus M helps in controlling the population of gnats; they manage to do this by killing the moth's larvae.
Though the virus is always present in the larvae, it is only every six or seven years that the virus seriously
decimates the numbers of larvae, greatly reducing the population of the gnats. Scientists are of the opinion
that the gnats, usually latent, are activated only when the larvae experience biological stress.

In case the above mentioned scientists are correct, it can be inferred that the decimation of gnat populations
by the virus M could be most probably activated by the following conditions?

A. A shift by the gnats from drought areas to a normal area occupied by them
B. The resultant stress from defoliation of trees attacked by the gnats for the second consecutive year.
C. Attacks on the larvae by all kinds by parasitic wasps and flies.
D. Starvation of the gnat larvae due to over population.
E. Spraying of gnat infested areas with laboratory - raised Virus M.

Ans : D

50. In a particular code, the digits from 0 to 9 inclusive are each represented by a different letter of the alphabet,
the letter always representing the same digit. In case the following sum
87
BOPB
+SKB
--------
CVBQ

holds true when it is expressed in digits, which of the following cannot be properly inferred:

A. B cannot be 0.
B. B must be less than 5.
C. Q must be even.
D. O + S must be greater than 8.
E. C must be greater than B by 1.

Ans : B

Questions 51 - 56

There are three on-off switches on a control panel A, B, and C. They have to be changed from an initial setting to a
second setting according to the following conditions : In case only switch A is the switch on in the initial setting , then
turn on switch B.

In case switches A and B are the only switches on in the initial setting, then turn on switch C. In case all the three
switches are on initially setting, then turn off the switch C. For any other initial setting, turn on all switches that are off
and turn off all switches, if any, that are on.

51. In case in the initial setting is the switches A and B are on and the switch C is off, then what could be the
second setting?
A. A on, B on, C on.
B. A on, B off, C on.
C. A on, B off, C off.
D. A off, B on, C off.
E. A off, B off, C on.

Ans : A

52. In case switch B is the only switch on in the initial setting, what must be the second setting?
A. A on, B on, C on.
B. A on, B on, C off.
C. A on, B off, C on.
D. A off, B off, C on.
E. A off, B off, C off.

Ans : C

53. In case all the three switches are on in the second setting, which among the following could have been the
initial setting ?
A. A on, B on, C on.
B. A on, B on, C off.
C. A on, B off, C on.
D. A on, B off, C off.
E. A off, B on, C off

Ans : B

54. In case switch A is off in the second setting, which among the following could have been the initial setting ?
A. A on, B on, C on.
B. A on, B on, C off.
C. A on, B off, C on.
D. A on, B off, C off.
E. A off, B on, C off.
88
Ans : C

55. In case only switch B is on in the second setting, which among the following could have been the initial
setting ?
A. A on, B on, C on.
B. A on, B off, C on.
C. A off, B on, C off.
D. A off, B off, C on.
E. A off, B off, C off.

Ans : B

56. Which among the following initial settings leads to a second setting, where only one switch is off ?
A. A on, B on, C off.
B. A on, B off, C on.
C. A off, B on, C on.
D. A off, B on, C off.
E. A off, B off, C off.

Ans : D

Questions 57 - 59

A bus has exactly six stops on its route. The bus first stops at stop one and then at stops two, three, four, five, and six
respectively. After the bus leaves stop six, the bus turns and returns to stop one and repeats the cycle. The stops are
at six building that are, in alphabetical order L, M, N, O, P, and Q.

P is the third stop.


M is the sixth stop.
The stop O is the stop immediately before Q.
N is the stop immediately before L.

57. In case N is the fourth stop, which among the following must be the stop immediately before P ?
A. O
B. Q
C. N
D. L
E. M

Ans : B

58. In case L is the second stop, which among the following must be the stop immediately before M ?
A. N
B. L
C. P
D. O
E. Q

Ans : E

59. In case a passenger gets on the bus at O, rides past one of the stops, and gets off at P, which of the
following must be true ?
A. O is stop one.
B. Q is stop three.
C. P is stop four.
D. N is stop five.
E. L is stop six.

Ans : A
89
60. A survey recently conducted revealed that marriage is fattening. The survey found that on an average,
women gained 23 pounds and men gained 18 pounds during 13 years of marriage. The answer to which
among the following questions would be the most appropriate in evaluating the reasoning presented in the
survey ?
A. Why is the time period of the survey 13 years, rather than 12 or 14 ?
B. Did any of the men surveyed gain less than 18 pounds during the period they were married ?
C. How much weight is gained or lost in 13 years by a single people of comparable age to those
studied in the survey ?
D. When the survey was conducted were the women as active as the men?
E. Will the gains seen in the study be retained over the lifetimes of the surveyed persons?

Ans : C

61. Classical works occupy a unique and peculiar position. They are imperishable as cultural landmarks, but the
views expressed in some of the works are often superseded by newer views. But the classics survive just
like ancient castles, as imposing features of the landscape, yet unsuited for habitation unless remodeled.

The principal point put forth in the above passage is that classical works.

A. Demand our respect and admiration even if their ideas are no longer current.
B. Prove to be obstacles in the path of new ideas as they are unduly respected.
C. Should be preserved and treasured as scholarly opinion keeps changing and they will come into
fashion again.
D. Lose their distinctive features as time passes.
E. Are not given due consideration by the younger generation.

Ans : A

62. Experts in the field of forestry are now questioning the policy of attempting to extinguish all forest fires
instead of allowing them to run their course and die out on their own. The study of forest fires indicates that
in the long run, forest fires may in fact, be beneficial to the ecology as a whole.

Among the following, which statement would logically follow from one of the ’observations’ referred
to in the passage above ?

A. Most of the forest fires are causes due to natural reasons like lightning rather than as a result of
human carelessness.
B. Every year the number of people dying as they attempt to fight forest fires is greater than the
number of people who would have died or suffered injury if the fires were allowed to run their own
course.
C. Accumulation of highly flammable dead brushwood and timber, which could lead to massive
uncontrollable fires is prevented by frequent small forest fires.
D. The animal and plant life destroyed by forest fires seldom represents endangered species.
E. Studies indicate that forest fires regularly occurred in the times prior to human occupation of forest
areas.

Ans : C

63. There is a theory that the dinosaurs inhabiting the earth were destroyed when a huge heavenly body hit the
earth. The impact of the heavenly body when it struck the earth threw a mass of pulverized debris into the
atmosphere. This dust created a fog, which blocked the sunlight depleting plant life. This resulted in the
perishing of the plant-eating dinosaurs; the meat-eating dinosaurs, who preyed on the plant-eaters inturn
starved to death.

Which among the following statements, would best support the above mentioned theory, if true?

A. Enough dust has been sent into the atmosphere by modern volcanic explosions to change the color
of sunsets around the world for several years.
B. Craters formed by heavenly bodies are plentiful in several areas of the South west where many
dinosaur fossils have also been found.
90
C. Studies conducted recently on the bone structure of dinosaurs show that they were very agile and
not ponderous awkward animals as popularly believed.
D. The sedimentary rock strata of the earth formed from atmospheric dust at the time the dinosaurs
disappeared contains minerals and other trace elements in proportions characteristic of the
heavenly bodies.
E. Many meat-and plant-eating species of mammals who coexisted with the dinosaurs continue to
exist today, although with a slightly modified appearance.

Ans : D

Questions 64 - 70

Five educational films A, B, C, D, & E are to be shown to a group of students. The films are to be shown in a
particular order, which conforms to the following conditions:

A must be shown earlier than C.


B must be shown earlier than D.
E should be the fifth film shown.

64. Which among the following is an acceptable order for showing the educational films ?
A. A, C, B, D, E
B. A, C, D, E, B
C. B, D, C, A, E
D. B, D, E, A, C
E. E, B, C, A, D

Ans : D

65. In case C is shown earlier than E, which among the following will hold true ?
A. A is the first film shown.
B. B is the second film shown.
C. C is the third film shown.
D. D is the fifth film shown.
E. E is the fourth film shown.

Ans : D

66. In case D is to be shown earlier than A, then for which among the following is there exactly one position from
first through fifth in which it can be scheduled to be shown ?
A. A
B. B
C. C
D. D
E. E

Ans : C

67. Which among the following is a pair of films that CANNOT both be shown earlier than E ?
A. A and B
B. A and D
C. B and C
D. B and D
E. C and D

Ans : E

68. In case D and E are shown as far apart from each other as possible, which among the following would be
true ?
A. A is shown earlier than B.
B. B is shown earlier than C.
C. C is shown earlier than E.
91
D. D is shown earlier than A.
E. E is shown earlier than B.

Ans : E

69. In case B, D and E are to be shown one after the other in the given order, the two positions from first to fifth
in which A could possibly be shown are
A. first and second.
B. first and fourth.
C. second and third.
D. third and fifth.
E. fourth and fifth.

Ans : B

70. In case exactly one film is shown between A and C, and exactly one film is shown between B and D, which
among the following will hold true ?
A. B is the film shown between A and C.
B. C is the film shown between B and D.
C. E is the film shown between A and C.
D. D is the last film shown.
E. E is the first film shown.

Ans : E

Analytical Section : Logical Reasoning

Directions :Each GRE sample logical reasoning question in this part of the assessment starts with a reading
passage containing the information to be used to choose between correct and incorrect logical conclusions. These
conclusions are based on the information in the passage. After this reading passage, you are given a lead-in phrase
that tells you to choose from among five different responses. These possible responses are generated by correctly or
incorrectly applying logical thought to the information in the passage at the beginning of the question. They can be
thought of as different ways of completing a sentence that begins with the lead-in phrase.

Each GRE sample logical reasoning reading passage is based on actual Bureau of Labor Statistics documents but is
not necessarily a completely accurate representation of BLS work. It is important that you accept every fact in the
reading passage as true, when you evaluate the response choices offered. You should use only the information in the
passage as the basis for accepting or rejecting any response choices. Be careful not to allow any "facts" that are not
clearly stated in the reading passage, or any outside knowledge you may have of the "facts", to influence your
thinking.

1. Testifying before the Senate committee investigating charges that cigarette manufacturers had manipulated
nicotine levels in cigarettes in order to addict consumers to their products, tobacco executives argued that
cigarette smoking is not addictive. The primary reason they gave in support of this claim was that cigarette
smoking was not regulated by the Federal Drug Administration.

For the tobacco executives' argument to be logically correct, which of the following must be assumed?

A. Substances that are not addictive are not regulated by ...........the Federal Drug Administration.
B. The tobacco executives lied when they claimed that ...........cigarette smoking was not addictive.
C. Some addictive substances are not regulated by the ...........Federal Drug Administration.
D. There is no scientific proof that cigarette smoking is ...........addictive.
E. Substances that are not regulated by the Federal Drug ...........Administration are not addictive.

Ans : E

2. People should be held accountable for their own behavior, and if holding people accountable for their own
behavior entails capital punishment, then so be it. However, no person should be held accountable for
behavior over which he or she had no control.

Which of the following is the most logical conclusion of the argument above?
92
A. People should not be held accountable for the ...........behavior of other people.
B. People have control over their own behavior.
C. People cannot control the behavior of other people.
D. Behavior that cannot be controlled should not be ...........punished.
E. People have control over behavior that is subject ...........to capital punishment.

Ans : B

3. There is clear evidence that the mandated use of safety seats by children under age four has resulted in
fewer child fatalities over the past five years. Compared to the five-year period prior to the passage of laws
requiring the use of safety seats, fatalities of children under age four have decreased by 30 percent.

Which one of the following, if true, most substantially strengthens the argument above?

A. The number of serious automobile accidents involving ...........children under age four has remained
steady over the ...........past five years.
B. Automobile accidents involving children have decreased ...........sharply over the past five years.
C. The use of air bags in automobiles has increased by ...........30 percent over the past five years.
D. Most fatal automobile accidents involving children under ...........age four occur in the driveway of
their home.
E. The number of teenage drivers has increased by 30 ...........percent over the past five years.

Ans : A

4. Lycopene, glutathione, and glutamine are powerful antioxidants that neutralize the free radicals that are
produced in the body as a result of routine bodily processes. An excess of these free radicals in your system
causes rapid aging because they accelerate the rate of cellular damage. Aging is simply the result of this
damage. Thus, to slow down aging it is necessary to supplement your diet with these antioxidants on a daily
basis.

Which of the following, if true, most seriously undermines the author's contention?

A. Most persons aren't concerned with the effects of aging ...........until it is too late to do anything.
B. Exercise associated with normal daily activities effectively ...........neutralizes and dissipates the free
radicals that are ...........produced as a result of routine bodily processes.
C. The cost of antioxidants is exorbitantly high and well ...........beyond the budget of most consumers.
D. Only overweight people who do not exercise on a daily ...........basis are likely to have an excess of
free radicals in their ...........systems.
E. Smoking cigarettes is one of the main causes of cellular ...........damage in humans.

Ans : B

5. Is it wrong for doctors to lie about their patients' illnesses? Aren't doctors just like any other people we hire to
do a job for us? Surely, we would not tolerate not being told the truth about the condition of our automobile
from the mechanic we hired to fix it, or the condition of our roof from the carpenter we employed to repair it.
Just as these workers would be guilty of violating their good faith contracts with us if they were to do this,
doctors who lie to their patients about their illnesses violate these contracts as well, and this is clearly wrong.

The conclusion of the argument is best expressed by which of the following?

A. Doctors who lie to their patients about their illnesses ...........violate their good faith contracts with
their patients.
B. Doctors often lie to their patients about their illnesses.
C. Doctors are just hired workers like mechanics and ...........carpenters.
D. It is wrong for doctors to lie about their patients' ...........illnesses.
E. Doctors, like mechanics and carpenters, enter into good ...........faith contracts with us when we hire
them.

Ans : D
93
6. As any economist knows, healthy people pose less of an economic burden to society than unhealthy people.
Not surprisingly, then, every dollar our state government spends on prenatal care for undocumented
immigrants will save taxpayers of this state three dollars.

Which of the following, if true, would best explain why the statistics cited above are not surprising?

A. The state's taxpayers pay for prenatal care of all ...........immigrants.


B. Babies born in this state to undocumented immigrant ...........parents are entitled to infant care
benefits from ...........the state.
C. State benefits for prenatal care serve to promote ...........undocumented immigration.
D. Babies whose mothers did not receive prenatal care ...........are just as healthy as other babies.
E. Pregnant women who do not receive prenatal care are ...........more likely to experience health
problems than ...........other pregnant women.

Ans : E

7. Beautiful beaches attract people, no doubt about it. Just look at this city's beautiful beaches, which are
among the most overcrowded beaches in the state.

Which of the following exhibits a pattern of reasoning most similar to the one exhibited in the argument
above?

A. Moose and bear usually appear at the same drinking ...........hole at the same time of day.
Therefore, moose and ...........bear must grow thirsty at about the same time.
B. Children who are scolded severely tend to misbehave ...........more often than other children. Hence
if a child is ...........not scolded severely that child is less likely to ...........misbehave.
C. This software program helps increase the work ...........efficiency of its users. As a result, these
users ...........have more free time for other activities.
D. During warm weather my dog suffers from fleas ...........more so than during cooler weather.
Therefore, ...........fleas must thrive in a warm environment.
E. Pesticides are known to cause anemia in some people. ...........However, most anemic people live
in regions where ...........pesticides are not commonly used.

Ans : D

8. Our school district should not spend its money on the new Verbal Advantage reading program. After all, our
students get all the reading practice they need by studying history and science.

The argument above depends on which the following assumptions?

A. The Verbal Advantage program would not help the ...........students learn history and science.
B. Other reading programs are just as effective but ...........less expensive than the Verbal Advantage
program.
C. The Verbal Advantage program involves only reading ...........practice.
D. Teaching students history and science is more ...........important than teaching them reading skills.
E. The students can already read well enough to ...........study history and science

Ans : C

9. A study of native born residents in Newland found that two-thirds of the children developed considerable
levels of nearsightedness after starting school, while their illiterate parents and grandparents, who had no
opportunity for formal schooling, showed no signs of this disability.

If the above statements are true, which of the following conclusions is most strongly supported by them?

A. Only people who have the opportunity for formal ...........schooling develop nearsightedness.
B. People who are illiterate do not suffer from ...........nearsightedness.
C. The nearsightedness in the children is caused by the ...........visual stress required by reading and
other class work.
D. Only literate people are nearsighted.
94
E. One-third of the children are illiterate.

Ans : C

10. Newspaper publishers earn their profits primarily from advertising revenue, and potential advertisers are
more likely to advertise in newspapers with a wide circulation—a large number of subscribers and other
readers—than with other newspapers. But the circulation of the newspaper that is currently the most
profitable one in this city has steadily declined during the last two years, while the circulation of one of its
competitors has steadily increased.

Any of the following, if true, would help explain the apparent discrepancy between the two statements above
EXCEPT:

A. Advertisers generally switch from the most widely ...........circulated newspaper to another one only
when the ...........other one becomes the most widely circulated ...........newspaper instead.
B. Advertising rates charged by the most profitable ...........newspaper in the city are significantly
higher than ...........those charged by its competitors.
C. The most profitable newspaper in the city receives ...........revenue from its subscribers as well from
advertisers.
D. The circulation of the most profitable newspaper ...........in the city is still greater than than of any of
its ...........competitors.
E. The number of newspapers competing viably with the ...........most profitable newspaper in the city
has increased ...........during the last two years.

Ans : E

11. Although most of the fastest growing jobs in today's economy will require a college degree, many of the new
jobs being created-from home health aide to desktop publisher-require knowledge other than that gained
from earning a degree. For workers in those jobs, good basic skills in reading, communigreion, and
mathematics play an important role in getting a job and developing a career.

From the information given above it can be validly concluded that, in today's economy,

A. skills in reading, communigreion, and mathematics play an important role in developing a career as
a desktop publisher
B. the majority of the new jobs being created require knowledge other than that gained from earning a
college degree
C. a job as a home health aide will rely more on communigreion skills than on basic skills in reading
and mathematics
D. if a job is one of the fastest growing jobs, it will require a college degree
E. desktop publisher jobs and home health aide jobs are not among the fastest growing jobs

Ans : A

12. According to the National Agricultural Aviation Society (NAAS), without the use of crop protection products
to control insects, weeds, and diseases, crop yields per acre will drop by more than 50 percent. The first
aerial appligreion of insecticide occurred in 1921, and it was a huge success. By contrast, in today's
economy all aircraft that are classified as aerial appligreors do more than just apply insecticide; today, they
also spread seed and apply fertilizer.

From the information given above it CANNOT be validly concluded that

A. in today's economy, if an aerial appligreor is used, then it will be able to spread seed and to apply
fertilizer
B. according to the NAAS, if crop yields per acre never drop by more than 50 percent, then crop
protection products have been used to control insects, weeds, and diseases
C. in today's economy, any aircraft that cannot be used to apply fertilizer cannot be classified as an
aerial appligreor
D. in 1921, if an aircraft was used for the appligreion of insecticide, then it was not also used to spread
seed
95
E. according to the NAAS, if crop yields per acre drop by more than 50 percent, then crop protection
products have not been used to control insects, weeds, and diseases.

Ans : E

13. No national productivity measures are available for underground industries that may exist but remain
unreported. On the other hand, at least some industries that are run entirely by self-employed industrialists
are included in national productivity measures.

From the information given above, it can be validly concluded that

A. there are at least some industries run entirely by self-employed industrialists that are underground
industries
B. no industries that are run entirely by self-employed industrialists operate underground
C. there are at least some industries other than those run entirely by self-employed industrialists that
are underground industries
D. there are at least some industries run entirely by self-employed industrialists that are not
underground industries
E. there are at least some underground industries for which national productivity measures are
available

Ans : D

14. Lou observes that if flight 409 is canceled, then the manager could not possibly arrive in time for the
meeting. But the flight was not canceled. Therefore, Lou concludes, the manager will certainly be on time.
Evelyn replies that even if Lou's premises are true, his argument is fallacious. And therefore, she adds, the
manager will not arrive on time after all.

Which of the following is the strongest thing that we can properly say about this discussion?

A. Evelyn is mistaken in thinking Lou's argument to be fallacious, and so her own conclusion is
unwarranted.
B. Evelyn is right about Lou's argument, but nevertheless her own conclusion is unwarranted.
C. Since Evelyn is right about Lou's argument, her own conclusion is well supported.
D. Since Evelyn is mistaken about Lou's argument, her own conclusion must be false.
E. Evelyn is right about Lou's argument, but nevertheless her own conclusion is false.

Ans : B

15. Sally has never received a violation from the Federal Aviation Administration during her 16-year flying
career. Sally must be a great pilot.

Which of the following can be said about the reasoning above?

A. The definitions of the terms create ambiguity.


B. The argument uses circular reasoning.
C. The argument works by analogy.
D. The argument is built upon hidden assumptions.
E. This is an example of an argument that is directed against the source of the claim rather than the
claim itself.

Ans : D

16. The Japanese economic model created strong domestic industries through subsidies from its Ministry of
Trade and by closing off competitive foreign firms to its domestic market. This strategy promised to help
economic growth by incubating domestic industries. New Japanese industries could count on a known local
demand and would be protected from competition by tariffs and other barriers. The program could reduce
the amount of imports and therefore improve the nation's balance of trade.

Which of the following, based on the passage above, is a weakness in this economic strategy?
96
A. A protectionist policy will create animosity among other nations.
B. Fast growth of small industries will create a class of millionaires and increase the inequality of
income.
C. Subsidies and import constraints keep domestic prices high and impose a burden on consumers.
D. Quotas are more regressive than tariffs.
E. The demand for the products made by the incubated industries would not be known.

Ans : C

17. Historically, famines have generally been followed by periods of rising wages, because when a labor force is
diminished, workers are more valuable in accordance with the law of supply and demand. The Irish potato
famine of the 1840s is an exception; it resulted in the death or emigration of half of Ireland's population, but
there was no significant rise in the average wages in Ireland in the following decade.

Which one of the following, if true, would LEAST contribute to an explanation of the exception to the
generalization?

A. Improved medical care reduced the mortality rate among able-bodied adults in the decade following
the famine to below prefamine levels.
B. Eviction policies of the landowners in Ireland were designed to force emigration of the elderly and
infirm, who could not work, and to retain a high percentage of able-bodied workers.
C. Advances in technology increased the efficiency of industry and agriculture, and so allowed
maintenance of economic output with less demand for labor.
D. The birth rate increased during the decade following the famine, and this compensated for much of
the loss of population that was due to the famine.
E. England, which had political control of Ireland, legislated artificially low wages to provide English-
owned industry and agriculture in Ireland with cheap labor.

Ans : D

18. Cars are safer than planes. Fifty percent of plane accidents result in death, while only one percent of car
accidents result in death.

Which of the following, if true, would most seriously weaken the argument above?

A. Planes are inspected more often than cars.


B. The number of car accidents is several hundred thousand times higher than the number of plane
accidents.
C. Pilots never fly under the influence of alcohol, while car drivers often do.
D. Plane accidents are usually the fault of air traffic controllers, not pilots.
E. Planes carry more passengers than cars do.

Ans : B

19. The body of anyone infected by virus X will, after a week, produce antibodies to fight the virus; the
antibodies will increase in number for the next year or so. There is now a test that reliably indigrees how
many antibodies are present in a person's body. If positive, this test can be used during the first year of
infection to estimate to within a month how long that person has had the virus.

Which one of the following conclusions is best supported by the statements above?

A. Antibodies increase in number only until they have defeated the virus.
B. Without the test for antibodies, there is no way of establishing whether a person has virus X.
C. Antibodies are produced only for viral infections that cannot be fought by any other body defenses.
D. If a person remains infected by virus X indefinitely, there is no limit to the number of antibodies that
can be present in the person's body.
E. Anyone infected by virus X will for a time fail to exhibit infection if tested by the antibody test.

Ans : E
97
20. Ever since I arrived at the college last week, I've been shocked by the poor behavior of the students. The
student population is completely lacking in proper social skills.

Which of the following, if true, would weaken the above conclusion?

A. Students who are away from their parents often exhibit rude behavior.
B. The college numbers over 50,000 students.
C. The narrator is a student and has interacted with many students.
D. Social skills should not be expected of college students.
E. The narrator was reluctant to stay at the college.

Ans : B

21. A study of a math program implemented in several pre-schools indigrees that children who received the
specialized Math Plus math edugreion between the ages three and five had significantly higher math scores
in 3rd and 4th grade than their classmates who did not receive this instruction. The proponents of the math
argue that the introduction of this program for all children age three to five will significantly improve their
chances for success in school.

Which of the following, if true, would most seriously weaken the above argument?

A. Most parents send their children to preschool for social development and do not have a clear idea
about what types of edugreion they want for their children.
B. Cognitive abilities of 3- to 5-year-old children are constantly changing.
C. The children in the pre-schools that were studied had previously been exposed to another math
enrichment program.
D. Children are not really interested in enrichment programs in preschool.
E. The cost factor needs to be specified and established before a large scale program can be
undertaken.

Ans : C

22. The symptoms of mental disorders are behavioral, cognitive, or emotional problems. Some patients with
mental disorders can be effectively treated with psychotherapy, but it is now known that in some patients'
mental disorders result from chemical imbalances affecting the brain. Thus, these patients can be effectively
treated only with medigreion that will reduce or correct the imbalance.

The argument depends on assuming which one of the following?

A. Treatment by psychotherapy can produce no effective reduction in or correction of chemical


imbalances that cause mental disorders.
B. Treatment with medigreion always shows faster results for patients with mental disorders than does
treatment with psychotherapy
C. Most mental disorders are not the result of chemical imbalances affecting the brain.
D. Medigreion is always more effective in treating patients with mental disorders than is
psychotherapy.
E. Treatment with psychotherapy has no effect on mental disorders other than a reduction of the
symptoms.

Ans : A

23. Dear Editor: I feel obliged to comment on the unfair review you published last week written by Robert
Duxbury. Your readers should know that Mr. Duxbury recently published his own book that covered the
same topic as my book, which you asked him to review. It is regrettable that Mr. Duxbury should feel the
need to belittle a competing work in the hope of elevating his own book.

The author of the letter above makes her point by employing which method of argument?

A. Attacking the motives of the author of the unfavorable review.


B. Attacking the book on the same topic written by the author of the review.
98
C. Contrasting her own book with that written by the author of the review.
D. Questioning the judgment of the author of the unfavorable review.
E. Stating that her book should not have been reviewed by the author of a competing work.

Ans : A

24. The government of Zunimagua has refused to schedule free elections, release political prisoners, or restore
freedom of speech; therefore, no more financial aid from the United States should be provided to
Zunimagua.

Which of the following is an assumption made in the argument above?

A. Withdrawal of U.S. aid from Zunimagua will force a change in the policies of its government.
B. The people of Zunimagua would be better off if their present despotic government were
overthrown.
C. The government of Zunimagua is dependent on continued U.S. aid for its existence.
D. U.S. aid should be given only to countries willing to adopt policies in line with U.S. interests and
goals.
E. U.S. aid should be withdrawn from any country that refuses to operate its government along
democratic lines.

Ans : E

25. Many people argue that the death penalty deters murder. However, the notorious killer Ned Grandy
deliberately moved to a state that imposes the death penalty just before embarking on a series of ferocious
murders. Thus, it seems clear that the existence of the death penalty does not serve as a deterrent to
murder.

The argument above may best be characterized as:

A. an appeal to emotion.
B. a flawed analogy.
C. a general conclusion based on a specific example.
D. circular reasoning.
E. an appligreion of a general principle to a specific example.

Ans : C

26. Steve and JoAnne are both members of a certain club, though they are not speaking to each other and
refuse to work with each other. Cecily, the club president, is appointing members to the fundraising
committee, but she has resolved that she will not appoint anyone without his or her explicit consent. Steve
tells Cecily, "I will not consent to appointment on that committee unless I know whether JoAnne is to be a
member of it." And JoAnne says, "I will not consent to be a member of that committee unless I know whether
Steve will be appointed to it."

If all three of these people stick by these resolutions, then:

A. Neither of them can be appointed to the committee.


B. The situation described in the scenario cannot arise, because it is inherently incoherent.
C. They must either both be appointed or both be left out.
D. The committee may finally have one of them, both of them, or neither of them as members.
E. Either one of them can be appointed, but not both.

Ans : E

27. Russia's aggressive fishing in the prime fishing grounds of the Northern Pacific has led to a sharp decline in
the populations of many fish and a general increase in the retail price of fish. This same pattern has
occurred with far too many of our scarce vital natural resources, resulting in high prices for many products. It
is likely then, that fish prices will continue to rise in the near future.

In making the argument above, the author relies on all of the following assumptions EXCEPT:
99
A. The scarcity of fish is a determining factor in its price.
B. The decline in the number of fish available will result in higher prices for fish in stores.
C. There will not be any substantial decrease in other costs involved in the fishing process that could
keep the price of fish from increasing.
D. Fish populations will not recover in the near future.
E. Fishing practices can substantially influence the demand for fish.

Ans : E

28. During the past year, Boz Corporation, a cigarette manufacturer, has engaged in a "corporate image"
advertising campaign. One executive now urges that the advertising be extended for another year because
profits have increased by 29% over the previous year. Another executive, however, is skeptical. She
observes that the increases are typical for the industry over the past year, although none of their competitors
have used corporate image advertising.

The most accurate way of summarizing the second executive's point would be:

A. She argues that the effect may not really be due to its supposed cause because there has not been
a sufficient lapse of time between the cause and the effect.
B. She argues that the assignment of a cause for this effect is premature, because there is as yet no
well-established theory of such interactions.
C. She argues that corporate image advertising is unprofitable, since it has evidently benefited
competitors as much as the corporation that paid for it.
D. She knows that effective advertising requires a constant influx of new ideas and approaches, and
she argues that one year of corporate image advertising is enough for awhile.
E. She argues that the effect may not be due to its alleged cause since the same effect is found
elsewhere without that cause.

Ans : E

29. The senate candidate expressed outrage that few judges have any background in technology, yet they try to
resolve cases involving high tech companies. He stated that not one federal judge has a degree or any
experience in computer technology.

A promising response to this concern, arguing that things are not as bad as they might seem, could involve
which of the following claims?

A. Most of the public policy questions in this area are really about the morality and the value of
scientific and technological developments. They do not require much technical understanding
beyond that of a layperson.
B. Computer scientists, by and large, have little interest in politics and public policy. It would be
difficult to find scientists with the degree of commitment required for a serious contribution to the
judicial system.
C. There is a lack of people who are qualified in both technical and legal areas of expertise.
D. There is very little opportunity for, and indeed little need for, technical expertise in the judicial
branch. There is therefore almost no way for a technical specialist to rise through the ranks to a
top-level position in government.
E. The rewards of a life as a judge, in terms of both money and prestige, are not high enough to
attract top-flight technical experts to this area.

Ans : A

30. There has been a sharp increase in the subscription prices of many professional and scholarly journals in
the past seven years. Many publishers ascribe the necessity for these increases to the easy availability of
photocopying facilities, which enable people simply to copy the articles they want rather than buying the
journal.

Which of the following, if it is true, would make this explanation more plausible?

A. The great majority of professional and scholarly journals have a massive backlog of papers
awaiting publigreion.
100
B. Over the past five years there has been a substantial decline in the number of individual
subscriptions to professional and scholarly journals, while library subscriptions have remained fairly
stable.
C. In the five years immediately preceding the price surge, there was a substantial decline in the
number of individual subscriptions to professional and scholarly journals, while library subscriptions
remained fairly stable.
D. Many libraries have recently begun cutting back on subscriptions to professional and scholarly
journals.
E. In almost every field, several new professional and scholarly journals have begun publigreion in the
past few years.

Ans : C

Analytical Section : Logical Reasoning

31. Smoking in bed has long been the main cause of home fires. Despite a significant decline in cigarette
smoking in the last two decades, there has been no comparable decline in the number of people killed in
home fires.

Each one of the following statements, if true over the last two decades, helps to resolve the apparent
discrepancy above EXCEPT:

A. Compared to other types of home fires, home fires caused by smoking in bed usually cause
relatively little damage before they are extinguished.
B. Home fires caused by smoking in bed often break out after the home's occupants have fallen
asleep.
C. Smokers who smoke in bed tend to be heavy smokers who are less likely to quit smoking than are
smokers who do not smoke in bed.
D. An increasing number of people have been killed in home fires that started in the kitchen.
E. Population densities have increased, with the result that one home fire can cause more deaths than
in previous decades.

Ans : B

32. Mrs. Mason is gifted with psychic powers that enable her to foretell future events. In the past, Mrs. Mason
has predicted such actual events as the election of President Clinton, the stock market crash of 1987, and
the St. Louis Cardinals' 1982 World Series victory. These are just a few of Mrs. Mason's accurate
predictions.

The answer to which of the following questions would be most useful in evaluating the strength of the
argument above?

A. What percentage of Mrs. Mason's predictions has come true?


B. Could the election of President Reagan have been predicted without the help of psychic powers?
C. What is the actual mechanism by which Mrs. Mason's psychic powers are supposed to operate?
D. How long before the events in question did Mrs. Mason make her accurate predictions?
E. Do most scientists accept the idea that the power to predict the future through psychic means really
exists?

Ans : A

33. An ingredient in coffee, known as RTC, has been found to inactivate common cold viruses in experiments. In
previous experiments, researchers found that inactivated common cold viruses can convert healthy cells into
cancer cells. It can be concluded that the use of coffee can cause cancer.

Which one of the following, if true, most seriously weakens the argument?

A. Several teams of scientists performed the various experiments, and all of the teams had similar
results.
B. The carcinogenic effect of RTC could be neutralized by the other ingredients found in coffee.
101
C. When RTC kills common cold viruses it weakens the immune system, and it might thus diminish
the body's ability to fight other viruses, including viruses linked to cancers.
D. If chemists modify the structure of RTC, RTC can be safely incorporated into medigreions to
prevent the common cold.
E. To lessen the undesirable side effects of chemotherapy, the use of coffee has been recommended
for cancer patients who are free of the common cold virus.

Ans : B

34. Jack Bygrave is an executive at a major South African diamond company that produces 2% of the world's
total annual diamond production. The CFO is anxious to maximize revenues and increase sales. Bygrave,
however, believes that increased production would only drive down the world price of diamonds and lower
revenues.

Which of the following represents the logical flaw in Bygrave's reasoning?

A. Jack connects the price of unrefined diamonds and the price of jewelry-quality diamonds.
B. He assumes that production goals are similar to financial goals.
C. He assumes that the supply produced by a single company can significantly alter the aggregate
supply for the market.
D. He assumes that seasonal and long term supply are proportional.
E. He correlates long-term and short-term demand.

Ans : C

35. The crux of creativity resides in the ability to manufacture variations on a theme. If we look at the history of
science, for instance, we see that every idea is built upon a thousand related ideas. Careful analysis leads
us to understand that what we choose to call a new theme or a new discovery is itself always and without
exception some sort of variation, on a deep level, of previous themes.

If all of the statements in the passage are true, each of the following must also be true EXCEPT:

A. A lack of ability to manufacture a variation on a previous theme connotes a lack of creativity


B. No scientific idea is entirely independent of all other ideas.
C. Careful analysis of a specific variation can reveal previous themes of which it is a variation.
D. All great scientific discoverers have been able to manufacture a variation on a theme.
E. Some new scientific discoveries do not represent, on a deep level, a variation on previous themes.

Ans : E

36. Studies of fatal automobile accidents reveal that, in the majority of cases in which one occupant of an
automobile is killed while another survives, it is the passenger, not the driver, who is killed. It is ironic that
the innocent passenger should suffer for the driver's carelessness, while the driver often suffers only minor
injuries or none at all.

Which of the following is an assumption underlying the reasoning in the passage above?

A. In most fatal automobile accidents, the driver of a car in which an occupant is killed is at fault.
B. Drivers of automobiles are rarely killed in auto accidents.
C. Most deaths in fatal automobile accidents are suffered by occupants of cars rather than by
pedestrians.
D. Auto safety experts should increase their efforts to provide protection for those in the passenger
seats of automobiles.
E. Automobile passengers sometimes play a contributing role in causing auto accidents.

Ans : A

37. The editors of Business Today magazine conducted a poll of its readers regarding the proposed increase in
the rate of income tax paid on profits from the sale of stocks. More than 60% of the readers opposed the
proposed tax. The editors announced that the majority of Americans opposed any increase in the tax on
profits from stock sales.
102
Which one of the following statements, if true, would most weaken the editor's conclusion?

A. Some readers of Business Today magazine are citizens of countries other than the United States.
B. Decisions concerning the income tax laws are made by the Congress rather than directly by the
people.
C. Most of those who earn profits from stock sales are wealthy and can afford to pay higher taxes.
D. The viewpoints of the vast majority of the readers of Business Today magazine differ from the
views of most Americans.
E. Not all readers of Business Today magazine responded to the editors' poll.

Ans : D

38. Several movie versions of Charles Dickens' Tale of Two Cities have been made. The original movie version
made in 1939 is the best because it is closest in spirit to the original novel.

An underlying assumption of the argument above is that a movie based on a novel should:

A. reflect the director's original interpretation of the main themes of the novel.
B. accurately depict the time and place in which the novel is set.
C. feature actors and actresses who closely resemble the characters in the novel both in body and
spirit.
D. faithfully render the details of the plot from the narrator's point of view.
E. capture the true meaning and intention of the novel.

Ans : E

You might also like